Предел стремящийся к 1. Решение пределов через раскрытие неопределённостей

Постоянное число а называется пределом последовательности {x n }, если для любого сколь угодно малого положительного числа ε > 0 существует номер N, что все значения x n , у которых n>N, удовлетворяют неравенству

|x n - a| < ε. (6.1)

Записывают это следующим образом: или x n → a.

Неравенство (6.1) равносильно двойному неравенству

a- ε < x n < a + ε, (6.2)

которое означает, что точки x n , начиная с некоторого номера n>N, лежат внутри интервала (a- ε, a+ ε), т.е. попадают в какую угодно малую ε-окрестность точки а .

Последовательность, имеющая предел, называется сходящейся , в противном случае - расходящейся .

Понятие предел функции является обобщением понятия предел последовательности, так как предел последовательности можно рассматривать как предел функции x n = f(n) целочисленного аргумента n .

Пусть дана функция f(x) и пусть a - предельная точка области определения этой функции D(f), т.е. такая точка, любая окрестность которой содержит точки множества D(f), отличные от a . Точка a может принадлежать множеству D(f), а может и не принадлежать ему.

Определение 1. Постоянное число А называется предел функции f(x) при x→ a, если для всякой последовательности {x n } значений аргумента, стремящейся к а , соответствующие им последовательности {f(x n)} имеют один и тот же предел А.

Это определение называют определением предел функции по Гейне, или “на языке последовательностей ”.

Определение 2 . Постоянное число А называется предел функции f(x) при x→ a, если, задав произвольное как угодно малое положительное число ε , можно найти такое δ >0 (зависящее от ε ), что для всех x , лежащих в ε-окрестности числа а , т.е. для x , удовлетворяющих неравенству
0 <
x-a < ε , значения функции f(x) будут лежать в ε-окрестности числа А, т.е. |f(x)-A| < ε.

Это определение называют определением предел функции по Коши, или “на языке ε - δ “.

Определения 1 и 2 равносильны. Если функция f(x) при x → a имеет предел , равный А, это записывается в виде

. (6.3)

В том случае, если последовательность {f(x n)} неограниченно возрастает (или убывает) при любом способе приближения x к своему пределу а , то будем говорить, что функция f(x) имеет бесконечный предел, и записывать это в виде:

Переменная величина (т.е. последовательность или функция), предел которой равен нулю, называется бесконечно малой величиной.

Переменная величина, предел которой равен бесконечности, называется бесконечно большой величиной .

Чтобы найти предел на практике пользуются следующими теоремами.

Теорема 1 . Если существует каждый предел

(6.4)

(6.5)

(6.6)

Замечание . Выражения вида 0/0, ∞/∞, ∞-∞ , 0*∞ , - являются неопределенными, например, отношение двух бесконечно малых или бесконечно больших величин, и найти предел такого вида носит название “раскрытие неопределенностей”.

Теорема 2. (6.7)

т.е. можно переходить к пределу в основании степени при постоянном показателе, в частности, ;

(6.8)

(6.9)

Теорема 3.

(6.10)

(6.11)

где e » 2.7 - основание натурального логарифма. Формулы (6.10) и (6.11) носят название первый замечательного предело и второй замечательный предел.

Используются на практике и следствия формулы (6.11):

(6.12)

(6.13)

(6.14)

в частности предел,

Eсли x → a и при этом x > a, то пишут x →a + 0. Если, в частности, a = 0, то вместо символа 0+0 пишут +0. Аналогично если x→ a и при этом xa-0. Числа и называются соответственно предел справа и предел слева функции f(x) в точке а . Чтобы существовал предел функции f(x) при x→ a необходимо и достаточно, чтобы . Функция f(x) называется непрерывной в точке x 0 , если предел

. (6.15)

Условие (6.15) можно переписать в виде:

,

то есть возможен предельный переход под знаком функции, если она непрерывна в данной точке.

Если равенство (6.15) нарушено, то говорят, что при x = x o функция f(x) имеет разрыв. Рассмотрим функцию y = 1/x. Областью определения этой функции является множество R , кроме x = 0. Точка x = 0 является предельной точкой множества D(f), поскольку в любой ее окрестности, т.е. в любом открытом интервале, содержащем точку 0, есть точки из D(f), но она сама не принадлежит этому множеству. Значение f(x o)= f(0) не определено, поэтому в точке x o = 0 функция имеет разрыв.

Функция f(x) называется непрерывной справа в точке x o , если предел

,

и непрерывной слева в точке x o, если предел

.

Непрерывность функции в точке x o равносильна ее непрерывности в этой точке одновременно и справа и слева.

Для того, чтобы функция была непрерывна в точке x o , например, справа, необходимо, во-первых, чтобы существовал конечный предел , а во-вторых, чтобы этот предел был равен f(x o). Следовательно, если хотя бы одно из этих двух условий не выполняется, то функция будет иметь разрыв.

1. Если предел существует и не равен f(x o), то говорят, что функция f(x) в точке x o имеет разрыв первого рода, или скачок .

2. Если предел равен +∞ или -∞ или не существует, то говорят, что в точке x o функция имеет разрыв второго рода .

Например, функция y = ctg x при x → +0 имеет предел, равный +∞ , значит, в точке x=0 она имеет разрыв второго рода. Функция y = E(x) (целая часть от x ) в точках с целыми абсциссами имеет разрывы первого рода, или скачки.

Функция, непрерывная в каждой точке промежутка , называется непрерывной в . Непрерывная функция изображается сплошной кривой.

Ко второму замечательному пределу приводят многие задачи, связанные с непрерывным ростом какой-либо величины. К таким задачам, например, относятся: рост вклада по закону сложных процентов, рост населения страны, распад радиоактивного вещества, размножение бактерий и т.п.

Рассмотрим пример Я. И. Перельмана , дающий интерпретацию числа e в задаче о сложных процентах. Число e есть предел . В сбербанках процентные деньги присоединяются к основному капиталу ежегодно. Если присоединение совершается чаще, то капитал растет быстрее, так как в образовании процентов участвует большая сумма. Возьмем чисто теоретический, весьма упрощенный пример. Пусть в банк положено 100 ден. ед. из расчета 100 % годовых. Если процентные деньги будут присоединены к основному капиталу лишь по истечении года, то к этому сроку 100 ден. ед. превратятся в 200 ден.ед. Посмотрим теперь, во что превратятся 100 ден. ед., если процентные деньги присоединять к основному капиталу каждые полгода. По истечении полугодия 100 ден. ед. вырастут в 100 × 1,5 = 150, а еще через полгода - в 150 × 1,5 = 225 (ден. ед.). Если присоединение делать каждые 1/3 года, то по истечении года 100 ден. ед. превратятся в 100 × (1 +1/3) 3 » 237 (ден. ед.). Будем учащать сроки присоединения процентных денег до 0,1 года, до 0,01 года, до 0,001 года и т.д. Тогда из 100 ден. ед. спустя год получится:

100 × (1 +1/10) 10 » 259 (ден. ед.),

100 × (1+1/100) 100 » 270 (ден. ед.),

100 × (1+1/1000) 1000 » 271 (ден. ед.).

При безграничном сокращении сроков присоединения процентов наращенный капитал не растет беспредельно, а приближается к некоторому пределу, равному приблизительно 271. Более чем в 2,71 раз капитал, положенный под 100% годовых, увеличиться не может, даже если бы наросшие проценты присоединялись к капиталу каждую секунду, потому что предел

Пример 3.1. Пользуясь определением предела числовой последовательности, доказать, что последовательность x n =(n-1)/n имеет предел, равный 1.

Решение. Нам надо доказать, что, какое бы ε > 0 мы ни взяли, для него найдется натуральное число N, такое, что для всех n N имеет место неравенство |x n -1| < ε.

Возьмем любое e > 0. Так как ; x n -1 =(n+1)/n - 1= 1/n, то для отыскания N достаточно решить неравенство 1/n< e . Отсюда n>1/ e и, следовательно, за N можно принять целую часть от 1/ e , N = E(1/ e ). Мы тем самым доказали, что предел .

Пример 3 .2 . Найти предел последовательности, заданной общим членом .

Решение. Применим теорему предел суммы и найдем предел каждого слагаемого. При n ∞ числитель и знаменатель каждого слагаемого стремится к бесконечности, и мы не можем непосредственно применить теорему предел частного. Поэтому сначала преобразуем x n , разделив числитель и знаменатель первого слагаемого на n 2 , а второго на n . Затем, применяя теорему предел частного и предел суммы, найдем:

.

Пример 3.3 . . Найти .

Решение. .

Здесь мы воспользовались теоремой о пределе степени: предел степени равен степени от предела основания.

Пример 3 .4 . Найти ().

Решение. Применять теорему предел разности нельзя, поскольку имеем неопределенность вида ∞-∞ . Преобразуем формулу общего члена:

.

Пример 3 .5 . Дана функция f(x)=2 1/x . Доказать, что предел не существует.

Решение. Воспользуемся определением 1 предела функции через последовательность. Возьмем последовательность { x n }, сходящуюся к 0, т.е. Покажем, что величина f(x n)= для разных последовательностей ведет себя по-разному. Пусть x n = 1/n. Очевидно, что , тогда предел Выберем теперь в качестве x n последовательность с общим членом x n = -1/n, также стремящуюся к нулю. Поэтому предел не существует.

Пример 3 .6 . Доказать, что предел не существует.

Решение. Пусть x 1 , x 2 ,..., x n ,... - последовательность, для которой
. Как ведет себя последовательность {f(x n)} = {sin x n } при различных x n → ∞

Если x n = p n, то sin x n = sin p n = 0 при всех n и предел Если же
x n =2
p n+ p /2, то sin x n = sin(2 p n+ p /2) = sin p /2 = 1 для всех n и следовательно предел . Таким образом, не существует.

Виджет для вычисления пределов on-line

В верхнем окошке вместо sin(x)/x введите функцию, предел которой надо найти. В нижнее окошко введите число, к которому стремится х и нажмите кнопку Calcular, получите искомый предел. А если в окне результата нажмете на Show steps в правом верхнем углу, то получите подробное решение.

Правила ввода функций: sqrt(x)- квадратный корень, cbrt(x) - кубический корень, exp(x) - экспонента, ln(x) - натуральный логарифм, sin(x) - синус, cos(x) - косинус, tan(x) - тангенс, cot(x) - котангенс, arcsin(x) - арксинус, arccos(x) - арккосинус, arctan(x) - арктангенс. Знаки: * умножения, / деления, ^ возведение в степень, вместо бесконечности Infinity. Пример: функция вводится так sqrt(tan(x/2)).

Математика — наука, строящая мир. Как учёный, так и простой человек — никто не сможет обойтись без неё. Сначала маленьких детей учат считать, потом складывать, вычитать, умножать и делить, к средней школе в ход вступают буквенные обозначения, а в старшей без них уже не обойтись.

Но сегодня речь пойдёт о том, на чём строится вся известная математика. О сообществе чисел под названием «пределы последовательностей».

Что такое последовательности и где их предел?

Значение слова «последовательность» трактовать нетрудно. Это такое построение вещей, где кто-то или что-то расположены в определённом порядке или очереди. Например, очередь за билетами в зоопарк — это последовательность. Причём она может быть только одна! Если, к примеру, посмотреть на очередь в магазин — это одна последовательность. А если один человек из этой очереди вдруг уйдёт, то это уже другая очередь, другой порядок.

Слово «предел» также легко трактуется — это конец чего-либо. Однако в математике пределы последовательностей — это такие значения на числовой прямой, к которым стремится последовательность чисел. Почему стремится, а не заканчивается? Всё просто, у числовой прямой нет конца, а большинство последовательностей, как лучи, имеют только начало и выглядят следующим образом:

х 1 , х 2 , х 3 , …х n …

Отсюда определение последовательности — функция натурального аргумента. Более простыми словами — это ряд членов некоторого множества.

Как строится числовая последовательность?

Простейший пример числовой последовательности может выглядеть так: 1, 2, 3, 4, …n…

В большинстве случаев для практических целей последовательности строятся из цифр, причём каждый следующий член ряда, обозначим его Х, имеет своё имя. Например:

х 1 — первый член последовательности;

х 2 — второй член последовательности;

х 3 — третий член;

х n — энный член.

В практических методах последовательность задаётся общей формулой, в которой есть некоторая переменная. Например:

Х n =3n, тогда сам ряд чисел будет выглядеть так:

Стоит не забывать, что при общей записи последовательностей можно использовать любые латинские буквы, а не только Х. Например: y, z, k и т. д.

Арифметическая прогрессия как часть последовательностей

Прежде чем искать пределы последовательностей, целесообразно поглубже окунуться в само понятие подобного числового ряда, с которым все сталкивались, будучи в средних классах. Арифметическая прогрессия — это ряд чисел, в котором разница между соседними членами постоянна.

Задача: «Пусть а 1 =15, а шаг прогрессии числового ряда d=4. Постройте первые 4 члена этого ряда»

Решение: а 1 = 15 (по условию) — первый член прогрессии (числового ряда).

а 2 = 15+4=19 — второй член прогрессии.

а 3 =19+4=23 — третий член.

а 4 =23+4=27 — четвёртый член.

Однако подобным методом трудно добраться до крупных значений, например до а 125. . Специально для таких случаев была выведена удобная для практики формула: а n =a 1 +d(n-1). В данном случае а 125 =15+4(125-1)=511.

Виды последовательностей

Большинство последовательностей бесконечны, это стоит запомнить на всю жизнь. Существует два интересных вида числового ряда. Первый задаётся формулой а n =(-1) n . Математики часто называют эту последовательностей мигалкой. Почему? Проверим её числовой ряд.

1, 1, -1 , 1, -1, 1 и т. д. На подобном примере становится ясно, что числа в последовательностях могут легко повторяться.

Факториальная последовательность. Легко догадаться — в формуле, задающей последовательность, присутствует факториал. Например: а n = (n+1)!

Тогда последовательность будет выглядеть следующим образом:

а 2 = 1х2х3 = 6;

а 3 = 1х2х3х4 =24 и т. д.

Последовательность, заданная арифметической прогрессией, называется бесконечно убывающей, если для всех её членов соблюдается неравенство -1

а 3 = - 1/8 и т. д.

Существует даже последовательность, состоящая из одного и того же числа. Так, а n =6 состоит из бесконечного множества шестёрок.

Определение предела последовательности

Пределы последовательностей давно существуют в математике. Конечно, они заслужили свое собственное грамотное оформление. Итак, время узнать определение пределов последовательностей. Для начала рассмотрим подробно предел для линейной функции:

  1. Все пределы обозначаются сокращённо lim.
  2. Запись предела состоит из сокращения lim, какой-либо переменной, стремящейся к определённому числу, нулю или бесконечности, а также из самой функции.

Легко понять, что определение предела последовательности может быть сформулировано следующим образом: это некоторое число, к которому бесконечно приближаются все члены последовательности. Простой пример: а x = 4x+1. Тогда сама последовательность будет выглядеть следующим образом.

5, 9, 13, 17, 21…x …

Таким образом, данная последовательность будет бесконечно увеличиваться, а, значит, её предел равен бесконечности при x→∞, и записывать это следует так:

Если же взять похожую последовательность, но х будет стремиться к 1, то получим:

А ряд чисел будет таким: 1.4, 1.8, 4.6, 4.944 и т. д. Каждый раз нужно подставлять число всё больше приближеннее к единице (0.1, 0.2, 0.9, 0.986). Из этого ряда видно, что предел функции — это пять.

Из этой части стоит запомнить, что такое предел числовой последовательности, определение и метод решения простых заданий.

Общее обозначение предела последовательностей

Разобрав предел числовой последовательности, определение его и примеры, можно приступить к более сложной теме. Абсолютно все пределы последовательностей можно сформулировать одной формулой, которую обычно разбирают в первом семестре.

Итак, что же обозначает этот набор букв, модулей и знаков неравенств?

∀ — квантор всеобщности, заменяющий фразы «для всех», «для всего» и т. п.

∃ — квантор существования, в данном случае обозначает, что существует некоторое значение N, принадлежащее множеству натуральных чисел.

Длинная вертикальная палочка, следующая за N, значит, что данное множество N «такое, что». На практике она может означать «такая, что», «такие, что» и т. п.

Для закрепления материала прочитайте формулу вслух.

Неопределённость и определённость предела

Метод нахождения предела последовательностей, который рассматривался выше, пусть и прост в применении, но не так рационален на практике. Попробуйте найти предел для вот такой функции:

Если подставлять различные значения «икс» (с каждым разом увеличивающиеся: 10, 100, 1000 и т. д.), то в числителе получим ∞, но в знаменателе тоже ∞. Получается довольно странная дробь:

Но так ли это на самом деле? Вычислить предел числовой последовательности в данном случае кажется достаточно легко. Можно было бы оставить всё, как есть, ведь ответ готов, и получен он на разумных условиях, однако есть ещё один способ специально для таких случаев.

Для начала найдём старшую степень в числителе дроби — это 1, т. к. х можно представить как х 1 .

Теперь найдём старшую степень в знаменателе. Тоже 1.

Разделим и числитель, и знаменатель на переменную в высшей степени. В данном случае дробь делим на х 1 .

Далее найдём, к какому значению стремится каждое слагаемое, содержащее переменную. В данном случае рассматриваются дроби. При х→∞ значение каждой из дробей стремится к нулю. При оформлении работы в писменном виде стоит сделать такие сноски:

Получается следующее выражение:

Конечно же, дроби, содержащие х, не стали нулями! Но их значение настолько мало, что вполне разрешено не учитывать его при расчётах. На самом же деле х никогда не будет равен 0 в данном случае, ведь на ноль делить нельзя.

Что такое окрестность?

Предположим, в распоряжении профессора сложная последовательность, заданная, очевидно, не менее сложной формулой. Профессор нашёл ответ, но подходит ли он? Ведь все люди ошибаются.

Огюст Коши в своё время придумал отличный способ для доказательства пределов последовательностей. Его способ назвали оперированием окрестностями.

Предположим, что существует некоторая точка а, её окрестность в обе стороны на числовой прямой равна ε («эпсилон»). Поскольку последняя переменная — расстояние, то её значение всегда положительно.

Теперь зададим некоторую последовательность х n и положим, что десятый член последовательности (x 10) входит в окрестность а. Как записать этот факт на математическом языке?

Допустим, х 10 находится правее от точки а, тогда расстояние х 10 -а<ε, однако, если расположить «икс десятое» левее точки а, то расстояние получится отрицательным, а это невозможно, значит, следует занести левую часть неравенства под модуль. Получится |х 10 -а|<ε.

Теперь пора разъяснить на практике ту формулу, о которой говорилось выше. Некоторое число а справедливо называть конечной точкой последовательности, если для любого её предела выполняется неравенство ε>0, причём вся окрестность имеет свой натуральный номер N, такой, что всё члены последовательности с более значительными номерами окажутся внутри последовательности |x n - a|< ε.

С такими знаниями легко осуществить решение пределов последовательности, доказать или опровергнуть готовый ответ.

Теоремы

Теоремы о пределах последовательностей — важная составляющая теории, без которой невозможна практика. Есть всего лишь четыре главных теоремы, запомнив которые, можно в разы облегчить ход решения или доказательства:

  1. Единственность предела последовательности. Предел у любой последовательности может быть только один или не быть вовсе. Тот же пример с очередью, у которой может быть только один конец.
  2. Если ряд чисел имеет предел, то последовательность этих чисел ограничена.
  3. Предел суммы (разности, произведения) последовательностей равен сумме (разности, произведению) их пределов.
  4. Предел частного от деления двух последовательностей равен частному пределов тогда и только тогда, когда знаменатель не обращается в ноль.

Доказательство последовательностей

Иногда требуется решить обратную задачу, доказать заданный предел числовой последовательности. Рассмотрим на примере.

Доказать, что предел последовательности, заданной формулой, равен нолю.

По рассмотренному выше правилу, для любой последовательности должно выполняться неравенство |x n - a|<ε. Подставим заданное значение и точку отсчёта. Получим:

Выразим n через «эпсилон», чтобы показать существование некоего номера и доказать наличие предела последовательности.

На этом этапе важно напомнить, что «эпсилон» и «эн» - числа положительные и не равны нулю. Теперь можно продолжать дальнейшие преобразования, используя знания о неравенствах, полученные в средней школе.

Откуда получается, что n > -3 + 1/ε. Поскольку стоит помнить, что речь идёт о натуральных числах, то результат можно округлить, занеся его в квадратные скобки. Таким образом, было доказано, что для любого значения окрестности «эпсилон» точки а=0 нашлось значение такое, что выполняется начальное неравенство. Отсюда можно смело утверждать, что число а есть предел заданной последовательности. Что и требовалось доказать.

Вот таким удобным методом можно доказать предел числовой последовательности, какой бы сложной она на первый взгляд ни была. Главное — не впадать в панику при виде задания.

А может, его нет?

Существование предела последовательности необязательно на практике. Легко можно встретить такие ряды чисел, которые действительно не имеют конца. К примеру, та же «мигалка» x n = (-1) n . очевидно, что последовательность, состоящая всего лишь из двух цифр, циклически повторяющихся, не может иметь предела.

Та же история повторяется с последовательностями, состоящими из одного числа, дробными, имеющими в ходе вычислений неопределённость любого порядка (0/0, ∞/∞, ∞/0 и т. д.). Однако следует помнить, что неверное вычисление тоже имеет место быть. Иногда предел последоватей найти поможет перепроверка собственного решения.

Монотонная последовательность

Выше рассматривались несколько примеров последовательностей, методы их решения, а теперь попробуем взять более определённый случай и назовём его «монотонной последовательностью».

Определение: любую последовательность справедливо называть монотонно возрастающей, если для нее выполняется строгое неравенство x n < x n +1. Также любую последовательность справедливо называть монотонной убывающей, если для неё выполняется неравенство x n > x n +1.

Наряду с этими двумя условиями существуют также подобные нестрогие неравенства. Соответственно, x n ≤ x n +1 (неубывающая последовательность) и x n ≥ x n +1 (невозрастающая последовательность).

Но легче понимать подобное на примерах.

Последовательность, заданная формулой х n = 2+n, образует следующий ряд чисел: 4, 5, 6 и т. д. Это монотонно возрастающая последовательность.

А если взять x n =1/n, то получим ряд: 1/3, ¼, 1/5 и т. д. Это монотонно убывающая последовательность.

Предел сходящейся и ограниченной последовательности

Ограниченная последовательность — последовательность, имеющая предел. Сходящаяся последовательность — ряд чисел, имеющий бесконечно малый предел.

Таким образом, предел ограниченной последовательности — это любое действительное или комплексное число. Помните, что предел может быть только один.

Предел сходящейся последовательности — это величина бесконечно малая (действительная или комплексная). Если начертить диаграмму последовательности, то в определённой точке она будет как бы сходиться, стремиться обратиться в определённую величину. Отсюда и название — сходящаяся последовательность.

Предел монотонной последовательности

Предел у такой последовательности может быть, а может и не быть. Сначала полезно понять, когда он есть, отсюда можно оттолкнуться при доказательстве отсутствия предела.

Среди монотонных последовательностей выделяют сходящуюся и расходящуюся. Сходящаяся — это такая последовательность, которая образована множеством х и имеет в данном множестве действительный или комплексный предел. Расходящаяся — последовательность, не имеющая предела в своём множестве (ни действительного, ни комплексного).

Причём последовательность сходится, если при геометрическом изображении её верхний и нижний пределы сходятся.

Предел сходящейся последовательности во многих случаях может быть равен нулю, так как любая бесконечно малая последовательность имеет известный предел (ноль).

Какую сходящуюся последовательность ни возьми, они все ограничены, однако далеко не все ограниченные последовательности сходятся.

Сумма, разность, произведение двух сходящихся последовательностей - также сходящаяся последовательность. Однако частное может быть также сходящейся, если оно определено!

Различные действия с пределами

Пределы последовательностей — это такая же существенная (в большинстве случаев) величина, как и цифры и числа: 1, 2, 15, 24, 362 и т. д. Получается, что с пределами можно проводить некоторые операции.

Во-первых, как и цифры и числа, пределы любых последовательностей можно складывать и вычитать. Исходя из третьей теоремы о пределах последовательностей, справедливо следующее равенство: предел суммы последовательностей равен сумме их пределов.

Во-вторых, исходя из четвёртой теоремы о пределах последовательностей, справедливо следующее равенство: предел произведения n-ого количества последовательностей равен произведению их пределов. То же справедливо и для деления: предел частного двух последовательностей равен частному их пределов, при условии что предел не равен нулю. Ведь если предел последовательностей будет равен нулю, то получится деление на ноль, что невозможно.

Свойства величин последовательностей

Казалось бы, предел числовой последовательности уже разобран довольно подробно, однако не раз упоминаются такие фразы, как «бесконечно маленькие» и «бесконечно большие» числа. Очевидно, если есть последовательность 1/х, где x→∞, то такая дробь бесконечно малая, а если та же последовательность, но предел стремится к нулю (х→0), то дробь становится бесконечно большой величиной. А у таких величин есть свои особенности. Свойства предела последовательности, имеющей какие угодно малые или большие величины, состоят в следующем:

  1. Сумма любого количества сколько угодно малых величин будет также малой величиной.
  2. Сумма любого количества больших величин будет бесконечно большой величиной.
  3. Произведение сколь угодно малых величин бесконечно мало.
  4. Произведение сколько угодно больших чисел — величина бесконечно большая.
  5. Если исходная последовательность стремится к бесконечно большому числу, то величина, ей обратная, будет бесконечно малой и стремиться к нулю.

На самом деле вычислить предел последовательности - не такая сложная задача, если знать простой алгоритм. Но пределы последовательностей — тема, требующая максимума внимания и усидчивости. Конечно, достаточно просто уловить суть решения подобных выражений. Начиная с малого, со временем можно достигнуть больших вершин.

Этот математический калькулятор онлайн поможет вам если нужно вычислить предел функции . Программа решения пределов не просто даёт ответ задачи, она приводит подробное решение с пояснениями , т.е. отображает процесс вычисления предела.

Данная программа может быть полезна учащимся старших классов общеобразовательных школ при подготовке к контрольным работам и экзаменам, при проверке знаний перед ЕГЭ, родителям для контроля решения многих задач по математике и алгебре. А может быть вам слишком накладно нанимать репетитора или покупать новые учебники? Или вы просто хотите как можно быстрее сделать домашнее задание по математике или алгебре? В этом случае вы также можете воспользоваться нашими программами с подробным решением.

Таким образом вы можете проводить своё собственное обучение и/или обучение своих младших братьев или сестёр, при этом уровень образования в области решаемых задач повышается.

Введите выражение функции
Вычислить предел

Обнаружено что не загрузились некоторые скрипты, необходимые для решения этой задачи, и программа может не работать.
Возможно у вас включен AdBlock.
В этом случае отключите его и обновите страницу.

У вас в браузере отключено выполнение JavaScript.
Чтобы решение появилось нужно включить JavaScript.
Вот инструкции, как включить JavaScript в вашем браузере .

Т.к. желающих решить задачу очень много, ваш запрос поставлен в очередь.
Через несколько секунд решение появится ниже.
Пожалуйста подождите сек...


Если вы заметили ошибку в решении , то об этом вы можете написать в Форме обратной связи .
Не забудте указать какую задачу вы решаете и что вводите в поля .



Наши игры, головоломки, эмуляторы:

Немного теории.

Предел функции при х->х 0

Пусть функция f(x) определена на некотором множестве X и пусть точка \(x_0 \in X \) или \(x_0 \notin X \)

Возьмем из X последовательность точек, отличных от х 0:
x 1 , x 2 , x 3 , ..., x n , ... (1)
сходящуюся к х*. Значения функции в точках этой последовательности также образуют числовую последовательность
f(x 1), f(x 2), f(x 3), ..., f(x n), ... (2)
и можно ставить вопрос о существовании ее предела.

Определение . Число А называется пределом функции f(х) в точке х = х 0 (или при х -> x 0), если для любой сходящейся к x 0 последовательности (1) значений аргумента x, отличных от x 0 соответствующая последовательность (2) значений функции сходится к числу A.


$$ \lim_{x\to x_0}{ f(x)} = A $$

Функция f(x) может иметь в точке x 0 только один предел. Это следует из того, что последовательность
{f(x n)} имеет только один предел.

Существует другое определение предела функции.

Определение Число А называется пределом функции f(x) в точке х = x 0 , если для любого числа \(\varepsilon > 0 \) существует число \(\delta > 0 \) такое, что для всех \(x \in X, \; x \neq x_0 \), удовлетворяющих неравенству \(|x-x_0| Используя логические символы, это определение можно записать в виде
\((\forall \varepsilon > 0) (\exists \delta > 0) (\forall x \in X, \; x \neq x_0, \; |x-x_0| Отметим, что неравенства \(x \neq x_0, \; |x-x_0| Первое определение основано на понятии предела числовой последовательности, поэтому его часто называют определением «на языке последовательностей». Второе определение называют определением «на языке \(\varepsilon - \delta \)».
Эти два определения предела функции эквивалентны и можно использовать любое из них в зависимости от того, какое более удобно при решении той или иной задачи.

Заметим, что определение предела функции «на языке последовательностей» называют также определением предела функции по Гейне, а определение предела функции «на языке \(\varepsilon - \delta \)» - определением предела функции по Коши.

Предел функции при x->x 0 - и при x->x 0 +

В дальнейшем будут использованы понятия односторонних пределов функции, которые определяются следующим образом.

Определение Число А называется правым (левым) пределом функции f(x) в точке x 0 , если для любой сходящейся к x 0 последовательности (1), элементы x n которой больше (меньше) x 0 , соответствующая последовательность (2) сходится к А.

Символически это записывается так:
$$ \lim_{x \to x_0+} f(x) = A \; \left(\lim_{x \to x_0-} f(x) = A \right) $$

Можно дать равносильное определение односторонних пределов функции «на языке \(\varepsilon - \delta \)»:

Определение число А называется правым (левым) пределом функции f(х) в точке x 0 , если для любого \(\varepsilon > 0 \) существует \(\delta > 0 \) такое, что для всех x, удовлетворяющих неравенствам \(x_0 Символические записи:

\((\forall \varepsilon > 0) (\exists \delta > 0) (\forall x, \; x_0

Тема 4.6.Вычисление пределов

Предел функции не зависит от того, определена она в предельной точке или нет. Но в практике вычисления пределов элементарных функций это обстоятельство имеет существенное значение.

1. Если функция является элементарной и если предельное значение аргумента принадлежит ее области определения, то вычисление предела функции сводится к простой подстановке предельного значения аргумента, т.к. предел элементарной функции f (x) при х стремящемся к а , которое входит в область определения, равен частному значению функции при х=а , т.е. lim f(x)=f(a ) .

2. Если х стремится к бесконечности или аргумент стремится к числу, которое не принадлежит области определения функции, то в каждом таком случае нахождение предела функции требует специального исследования.

Ниже приведены простейшие пределы, основанные на свойствах пределов, которые можно использовать как формулы:

Более сложные случаи нахождения предела функции:

рассматриваются каждый в отдельности.

В этом разделе будут приведены основные способы раскрытия неопределенностей.

1. Случай, когда при х стремящемся к а функция f (x) представляет отношение двух бесконечно малых величин

а) Сначала нужно убедится, что предел функции нельзя найти непосредственной подстановкой и при указанном изменении аргумента она представляет отношение двух бесконечно малых величин. Делаются преобразования, чтобы сократить дробь на множитель, стремящийся к 0. Согласно определению предела функции аргумент х стремится к своему предельному значению, никогда с ним не совпадая.

Вообще если ищется предел функции при х стремящемся к а , то необходимо помнить, что х не принимает значения а , т.е. х не равен а.

б) Применяется теорема Безу. Если ищется предел дроби, числитель и знаменатель которой многочлены, обращающиеся в 0 в предельной точке х=а , то согласно вышеназванной теореме оба многочлена делятся без остатка на х-а .

в) Уничтожается иррациональность в числителе или в знаменателе путем умножения числителя или знаменателя на сопряженное к иррациональному выражение, затем после упрощения дробь сокращается.

г) Используется 1-й замечательный предел (4.1).

д) Используется теорема об эквивалентности бесконечно малых и следующие б.м.:

2. Случай, когда при х стремящемся к а функция f (x) представляет отношение двух бесконечно больших величин

а) Деление числителя и знаменателя дроби на наивысшую степень неизвестного.

б) В общем случае можно использовать правило

3. Случай, когда при х стремящемся к а функция f (x) представляет произведение бесконечно малой величины на бесконечно большую

Дробь преобразовывается к виду, числитель и знаменатель которой одновременно стремятся к 0 или к бесконечности, т.е. случай 3 сводится к случаю 1 или случаю 2.

4. Случай, когда при х стремящемся к а функция f (x) представляет разность двух положительных бесконечно больших величин

Этот случай сводится к виду 1 или 2 одним из следующих способов:

а) приведение дробей к общему знаменателю;

б) преобразование функции к виду дроби;

в) избавление от иррациональности.

5. Случай, когда при х стремящемся к а функция f (x) представляет степень, основание которой стремится к 1, а показатель к бесконечности.

Функция преобразовывается таким образом, чтобы использовать 2-й замечательный предел (4.2).

Пример. Найти .

Так как х стремится к 3 , то числитель дроби стремится к числу 3 2 +3 *3+4=22, а знаменатель- к числу 3+8=11. Следовательно,

Пример

Здесь числитель и знаменатель дроби при х стремящемся к 2 стремятся к 0 (неопределенность вида), разложим числитель и знаменатель на множители, получим lim(x-2)(x+2)/(x-2)(x-5)

Пример

Умножим числитель и знаменатель на выражение, сопряженное к числителю, имеем

Раскрываем скобки в числителе, получим

Пример

Уровень 2. Пример. Приведем пример применения понятия предела функции в экономических расчетах. Рассмотрим обыкновенную финансовую сделку: предоставление в долг суммы S 0 с условием, что через период времени T будет возвращена сумма S T . Определим величину r относительного роста формулой

r=(S T -S 0)/S 0 (1)

Относительный рост можно выразить в процентах, умножив полученное значение r на 100.

Из формулы (1) легко определить величину S T :

S T = S 0 (1 + r )

При расчете по долгосрочным кредитам, охватывающим несколько полных лет, используют схему сложных процентов. Она состоит в том, что если за 1-й год сумма S 0 возрастает в (1 + r ) раз, то за второй год в (1 + r ) раз возрастает сумма S 1 = S 0 (1 + r ), то есть S 2 = S 0 (1 + r ) 2 . Аналогично получается S 3 = S 0 (1 + r ) 3 . Из приведенных примеров можно вывести общую формулу для вычисления роста суммы за n лет при расчете по схеме сложных процентов:

S n = S 0 (1 + r ) n .

В финансовых расчетах применяются схемы, где начисление сложных процентов производится несколько раз в году. При этом оговариваются годовая ставка r и количество начислений за год k . Как правило, начисления производятся через равные промежутки времени, то есть длина каждого промежутка T k составляет часть года. Тогда для срока в T лет (здесь T не обязательно является целым числом) сумма S T рассчитывается по формуле

(2)

где - целая часть числа, которая совпадает с самим числом, если, например, T ? целое число.

Пусть годовая ставка равна r и производится n начислений в год через равные промежутки времени. Тогда за год сумма S 0 наращивается до величины, определяемой формулой

(3)

В теоретическом анализе и в практике финансовой деятельности часто встречается понятие “непрерывно начисляемый процент”. Чтобы перейти к непрерывно начисляемому проценту, нужно в формулах (2) и (3) неограниченно увеличивать соответственно, числа k и n (то есть устремить k и n к бесконечности) и вычислить, к какому пределу будут стремиться функции S T и S 1 . Применим эту процедуру к формуле(3):

Заметим, что предел в фигурных скобках совпадает со вторым замечательным пределом. Отсюда следует, что при годовой ставке r при непрерывно начисляемом проценте сумма S 0 за 1 год наращивается до величины S 1 * , которая определяется из формулы

S 1 * = S 0 e r (4)

Пусть теперь сумма S 0 предоставляется в долг с начислением процента n раз в год через равные промежутки времени. Обозначим r e годовую ставку, при которой в конце года сумма S 0 наращивается до величины S 1 * из формулы (4). В этом случае будем говорить, что r e - это годовая ставка при начислении процента n раз в год, эквивалентная годовому проценту r при непрерывном начислении. Из формулы (3) получаем

S* 1 =S 0 (1+r e /n) n

Приравнивая правые части последней формулы и формулы (4), полагая в последней T = 1, можно вывести соотношения между величинами r и r e :

Эти формулы широко используются в финансовых расчётах.

2011 год Виосагмир И.А. Предел функции 2011 год Высшая математика для чайников. Предел функции [email protected] Высшая математика для чайников. Предел функции 2011 год 1 Пределфункции Введение Ну что же… Я приветствую Вас в своей первой книге, посвященной пределам функции. Это первая часть из моей будущей серии “высшая математика для чайников”. Название книги уже должно Вам многое о ней рассказать, но Вы его можете совершенно не так понять. Эта книга посвящена не “чайникам”, а всем тем, кому нелегко понять то, что творят профессоры в своих книгах. Я уверен, что Вы меня понимаете. Я сам находился и нахожусь в такой ситуации, что просто вынужден прочитывать одно и то же предложение несколько раз. Это нормально? Я думаю – нет. Так чем же моя книга отличается от всех других? Во-первых, здесь нормальный язык, а не “заумный”; во-вторых здесь разобрана масса примеров, которая, кстати, наверняка, пригодится вам; в-третьих, текст имеет существенное различие между собой – главные вещи выделены определенными маркерами, и наконец, моя цель лишь одна – ваше понимание. От Вас требуется только одного: желания и умения. “Умения?” – спросите Вы. Да! Умения и. Вообще рекомендуется завести отдельно тетрадку листов этак на 65, и все в ней писать. Все, что написано в этой книге. Результат будет впечатляющим, это я Вам обещаю. Так же лучше пользоваться разноцветными фломастерами. Ну что же, господа… Я хочу Вам пожелать успехов и понимания. Если Вы добьете эту книгу, Вы сможете многое!!! В моей книге будут встречаться некоторые обозначения. Крайне рекомендую им следовать. - учить обязательно! - рекомендуется попробовать сделать самим. - можно не учить, но нужно понять! Высшая математика для чайников. Предел функции 2011 год 2 Содержание Предел функции в точке………………………………………………………………………………………………….3 Теоремы о пределах………………………………………………………………………………………………………..13 Односторонние пределы………………………………………………………………………………………………..14 Предел при →∞…………………………………………………………………………………………………………..17 Бесконечно большие функции…………………………………………………………………………………………25 Графики элементарных функций…………………………………………………………………………………..26 Непрерывность функции в точке………………………………………………………………………………….31 Непрерывность сложной функции………………………………………………………………………………..33 Классификация точек разрыва………………………………………………………………………………………36 Непрерывность элементарных функций………………………………………………………………………41 Первый замечательный предел……………………………………………………………………………………..42 Второй замечательный предел……………………………………………………………………………………..47 Кратко о Maple………………………………………………………………………………………………………………..52 Сравнение бесконечно малых функций…………………………………………………………………………..55 Свойства символа “o малое”…………………………………………………………………………………………..60 Асимптотические формулы……………………………………………………………………………………………64 Правило Лопиталя……………………………………………………………………………………………………………72 Разложение в ряд Тейлора. Часть 1………………………………………………………………………………..80 Разложение в ряд Тейлора. Часть 2………………………………………………………………………………..88 Высшая математика для чайников. Предел функции 2011 год 3 Глава 1. Предел функции. Пусть числовая переменная величина, область ее изменения. Если каждому числу ∈ поставлено в соответствие некоторое число, то говорят, что на множестве определена функция, и пишут. Надеюсь это Вам понятно, но я на всякий случай поясню. Множество в данном случае – плоскость, состоящая из двух координатных осей – 0X и 0Y. Это вам должно быть известно еще со школы. Если Вы забыли это, открывайте класс 7 – 8 и повторяйте. Для примера, на рис. 1 изображена функция. Оси 0X и 0Y образуют область ее изменения. Мы прекрасно видим на рис. 1, как ведет себя функция. В таком случае говорят, что на множестве определена функция. Совокупность всех частных значений функции называется множеством значений. Другими словами, множество значений – это промежуток по оси OY, где определена функция. Для примера, рассмотрим рис. 1. – отсюда сразу видно, что 0, т.к. 0. На рисунке это явно видно. В данном случае область значений 0;∞. Запомните, множество значений смотрим по 0Y! Совокупность всех называется областью определения. Делаем вывод из предыдущих соображений и понимаем, что множество определений смотрим по 0. В нашем случае ОДЗ = ∞;∞. Точка ∈ или называется предельной точкой множества, если в любой окрестности точки имеются точки множества, отличные от. Здесь я дополнять ничего не буду. И так все ясно. Можно лишь добавить, что в нашем случае предельная точка множества области определения функции. Содержание: 1) Предел функции в точке 2) Теоремы о пределах 3) Односторонние пределы 4) Предел, при →∞ 5) Бесконечно большие функции 6) Графики элементарных функций 1.Предел функции в точке. Рис. 1 независимая переменная (аргумент). область определения функции. частное значение функции в точке. Высшая математика для чайников. Предел функции 2011 год 4 Так, давайте перед определением я в общих словах объясню, что такое предел функции. Число b, к которому стремится функция при стремлении x к числу, называется пределом функции. Вот так это все записывается: lim → Например, . Нам нужно узнать, к чему стремится (не равна!) функция, при →2. Сначала запишем предел: lim → lim → Теперь пришло время взглянуть на график. Проведем параллельно 0 линию через точку 2 на оси 0. Она пересекла наш график в точке 2;4. Опустим из этой точки на ось 0 перпендикуляр и… опа! Какое там значение? Все правильно, 4. Вот к чему стремится наша функция, при →2. Сложно? Ну, нет, конечно! Вы, наверное, заметили, что если подставить в функцию значение 2, то ответ будет таким же. Совершенно верно. Так и решаются эти “сложные” лимиты. Не забывайте проверять на определенность! Определенность, это, когда у нас есть понятный результат. Неопределенность, когда нет понятного результата. Например: или – все это неопределенность. Это очень важно, никогда не забывайте про это! Следовательно, у Вас должна быть в тетради вот такая запись (не забудьте нарисовать и рисунок): lim → lim → 2 4 Ну, с этим, в общем, все понятно. Потренируйтесь и посчитайте вот такие вот пределы: lim → ! 1 #;lim → ;lim → ;lim → √ То же самое происходит и для случая, когда →∞ или к другому бесконечному числу: lim → ∞ ∞ А вот пример, где есть неопределенность: lim → sin Если мы подставим под значение, равное 0, то вот, что у нас получится: . А это неопределенность, следовательно, решать мы не имеем права! Потом я Вас научу, как раскрывать неопределенность. Сейчас же вы должны не забывать про это. Подставили и проверили. Решается? Значит – определенность. Не решается? Ну что же, тогда потом решите. Когда все пройдете. Давайте перейдем к формальностям, то есть к определениям. Высшая математика для чайников. Предел функции 2011 год 5 Н Е О П Р Е Д Е Л Е Н Н О С Т Ь, 0 , 1 , ∞ , 0 ∙ ∞ , ∞ ∞ Определение 1 (предел функции по Коши) №1. Доказать, что lim → sin0. Для удобства, давайте сформулируем теорему (по Коши) для нашего случая. Вот, что у нас получится: Воспользуемся неравенством | sin | (| | ∀. Зададим произвольное * 0 и положим +*. Тогда если | | ,+, то | sin | (| | ,+*. Это и означает (согласно определению функции по Коши), что lim → sin0. По этому поводу в принципе объяснить нечего. Что касается | sin | (| | это просто нужно запомнить. Что касается * это очень маленькое число, находящееся в окрестности. №2. С помощью “* +” – рассуждений доказать, что lim → 4. Заполнить следующую таблицу: * 0.1 0.01 0.001 0.0001 … + Число b называется пределом функции в точке (при →), если ∀ 0 ∃ 0 такое, что ∀ , удовлетворяющего условиям, 0 | | , выполняется неравенство | | . Число 0 называется пределом функции sin в точке 0 (при → 0), если ∀ 0 ∃ 0 такое, что ∀ , удовлетворяющего условиям, 0 | | , выполняется неравенство | sin | . Высшая математика для чайников. Предел функции 2011 год 6 Пусть * 0 произвольно. Тогда | 4 | | 2 4 2 | (| 2 | 4 | 2 | (*, как только 0, | 2 | , √ 4 * 2 √ . Последнее неравенство тем более будет выполняться, если * √ 4 * 2 * 2 √ 4 * * 2 √ 4 4* * * 22 * + * | 2 | . Так, давайте все-таки рассмотрим этот пример более подробно. 1) Распишем определение: Число 4 называется пределом функции в точке 2 (при →2), если ∀* 0 ∃+ 0 такое, что ∀, удовлетворяющего условиям 0, 0, | 2 | ,+, выполняется неравенство | 4 | ,*. 2) Упростим: a) Условие: 0, | 2 | ,+ +, 2,+ 2 +,2 + b) неравенство: | 4 | ,* *, 4,* 4 *,4 * 3) Поймем: Число 4 называется пределом функции в точке 2 (при → 2), если ∀* 0 ∃+ 0 такое, что ∀, удовлетворяющего условиям 0, 2 +,2 +, выполняется неравенство 4 *,4 *. Все! Прочтите последнее определение, которое мы написали, используя график. Верно? Ну конечно верно! Этот способ я написал специально для вас, для понимания. Ни в какой литературе вы такого не найдете. Поэтому, если хотите по-настоящему все это быстро решать – пожалуйста! Да, объяснить, как это делается аналитически, я не Высшая математика для чайников. Предел функции 2011 год 7 уверен, что смогу. Пример я вам написал, теперь вы должны в нем сами разобраться, используя мой графический способ. Все строится от понимания, господа. Сейчас попробую объяснить все на аналитическом уровне. №3. Для закрепления. Доказать, используя определение Коши предела функции, что lim → −16 −4 = 2 Шаг 1: Зададим функцию () , которая является у нас выражением, стоящим у нас под знаком предела: = −16 −4 Поскольку мы рассматриваем предел, стремящийся к 4, нужно рассмотреть некоторую окрестность 4-ки, которая для данной функции определена. Например, интервал от 2 до 5. 40(2,5) Но! Заметьте, что функция у нас определена не всюду! Она не определена в 0 и при = 4. Надеюсь, Вы это понимаете, но на всякий случай распишу: −4 ≠ 0 → −4 ≠ 0 → 2 ≠ 0 ≠ 4 . Надеюсь все понятно. Так, отвлеклись, так что быстро идем дальше. Мы можем в принципе рассмотреть любой интервал, но нам такой удобнее 40(2,5). Шаг 2: Запишем определение предела функции () по Коши. ∀* > 0,∃+ > 0:∀ ≠ 4, | −4 | < + ⇒ | −2 | < * Это значит: для любого * мы должны найти такое+, что как только x у нас отлично от 4 и x-4 по модулю не превосходит + ⇒ | −2 | должно не превосходить*. Шаг 3: Преобразуем выражение | −2 | , ≠ 4. Высшая математика для чайников. Предел функции 2011 год 8 | −2 | = 3 −16 −4 −23 = 4 +4 −2 4 = | −4 | Эти преобразования нетрудно проделать самостоятельно. Надеюсь, у вас не вызывает это трудности. Итак, ∀* > 0,∃+ > 0:∀ ≠ 4, | −4 | < + ⇒ | −2 | < * и | −2 | = | | . Заметьте, информации все больше и больше! Шаг 4: Оценим сверху выражение | −2 | , ≠ 4, ∈ (2,5). 3 −16 −4 −23 < | −4 | 2 Поняли? Мы оцениваем | | , т.к. 5 −2 5 = | | . Следовательно, | | > | | . Здесь самое главное не запутаться. ∈ 2,5 −это условие мы поставили еще в начале. Отсюда идет сравнение дробей. Что больше | | или | | , где ∈ 2,5 . Конечно первая дробь. Где знаменатель меньше, там дробь больше (при одинаковых числителях). Шаг 5: Зададим + = 2*. Здесь мы можем брать и просто *, може взять и 5*. В данном случае нам удобнее всего, когда + = 2*. Итак, вот что мы сейчас имеем: ∀0 2,5 0 < | −4 | < + | −2 | < + 2 = * Вывод: Все! Мы доказали, что предел равен 2. Вывод один: если хотите решать все это, берите еще раз и решайте. И так до тех пор, пока не поймете. Я попытался описать, как это доказывается аналитически. Можете посмотреть на это все и с графической точки зрения, не забыв все упростить. Информация: Вообще, честно говоря, от Вас таких доказательств не должны требовать. Они слишком уж “плавающие”. Если Вам все же интересна эта тема, откройте любой Высшая математика для чайников. Предел функции 2011 год 9 учебник и посмотрите там материал. Соответственно, Вы ничего не поймете, если не напишете собственноручно решение + графики. Это Вам небольшая подсказка. Нарисуйте! И все сразу станет ясно. №1. Я забегаю немного вперед, но хотелось бы решить этот предел: lim → 16 4 Если мы подставим 4 под, у нас получится неопределенность: lim → 16 4 7 00 8 неопределенность! Что делать? Все просто. А давайте ка упростим дробь! 16 4 4 4 4 4 Все! Теперь, если мы подставим 4, у нас будет определенность, а, следовательно, мы можем решать. lim → 16 4 lim → 4 7 84 8 2 Вывод: от неопределенности мы избавляемся с помощью преобразований. №2. Посчитать предел: lim → 4 6 16 Здесь все очень просто. Разложим на множители числитель и знаменатель. Рассказываю первый и последний раз, как это делать. Что бы разложить знаменатель на множители, мы должны приравнять его к нулю и просто решить уравнение. Давайте сделаем это. 6 160 Что бы решить квадратное уравнение, прежде всего нужно найти дискриминант по формуле: D 4E Высшая математика для чайников. Предел функции 2011 год 10 ,E − элементы квадратного уравнения. В общем виде квадратное уравнение выглядит так: + +E = 0 Следовательно, в нашем случае = 1, = 6,E = −16. Подставляем значения и находим дискриминант: D = 36 +4 ∙ 1 ∙ 16 = 100 Далее находим корни квадратного уравнения, используя формулу, = − ± √ D 2 Подставляем и получаем: , = −6 ±10 2 = F = −6 +10 2 = 2 = −6 −10 2 = −8 Корни нашли, а значит мы очень близки к разложению на множители квадратного многочлена. Сначала запишем формулу: + +E = (−)(−) Заметим, что не всякий многочлен можно так расписать. В данном случае у нас нет никаких противоречий, и, следовательно, это можно делать. Таким образом: +6 −16 = (−2)(+8) Вот эту вещь вы должны уметь делать очень быстро. Ну, максимум – минута. Так что, если есть проблемы, сразу же их решайте. В числителе можно тоже разложить на множители. Это сделать гораздо проще, так как там разность квадратов. Напоминаю формулу: − = (−)(+) Таким образом: −4 = (−2)(+2) И получаем наш предел: lim → −4 +6 −16 = lim → (−2)(+2) (−2)(+8) = lim → (− 2) (+2) (− 2) (+8) = lim → +2 +8 = 4 10 = 25 Как видите, в общем-то решение в одну строчку. №3. Посчитать предел: Высшая математика для чайников. Предел функции 2011 год 11 lim → +5 +4 2 + −1 = lim → (+1)(+4) (2 −1)(+1) = lim → (+ 1) (+4) (2 −1)(+ 1) = lim → +4 2 −1 =− 33 = −1 №4. Посчитать предел: lim → − +2 −5 +3 +4 −7 +2 Здесь я вас хочу научить одной хитрой штучке. Как разложить на множители многочлен, у которого степень > 2? По дискриминанту мы этого делать не можем – он только для квадратных уравнений. Так что же делать? Объясняю: что бы разложить наш числитель на множители, нам достаточно найти хотя бы один корень. В данном случае нам ничего не остается делать, как подбирать. − +2 −5 +3 = 0 Когда равенство верно? Немного подумав, мы отвечаем: когда = 1. Верно? Подставьте 1 в уравнение и вы убедитесь в этом. Далее мы имеем право разложить наш многочлен на множители: − +2 −5 +3 = (−1) ∙ G() G − функция, которую нам предстоит найти. Решаем уравнение относительно G(). Получаем: G = − +2 −5 +3 −1 Ну а теперь просто делим одно на другое в столбик! − − + 2 − 5 + 3 − 1 − + 2 − 3 = G () − 2 − 5 + 3 2 − 2 − − 3 + 3 − 3 + 3 0 Таким образом, наша функция раскладывается так: − +2 −5 +3 = (−1) ∙ (+2 −3) То же самое делаем с знаменателем и получаем: +4 −7 +2 = (−1)(+5 −2) Высшая математика для чайников. Предел функции 2011 год 12 Итого: lim → 2 5 3 4 7 2 lim → 1 2 3 1 5 2 lim → 2 3 5 2 1 2 3 1 5 2 04 0 №5. Посчитать предел: lim → sin cos tg 1 lim → sin cos sin cos cos cos lim → sin cos sin cos cos lim → sin cos cos sin cos lim → cos √ 2 2 Определение 2 (предел функции по Гейне) Предел функции по Гейне редко можно встретить где-нибудь в практике. От Вас требуется лишь одно – выучить его на всякий случай. Может быть и пригодится. Подчеркнем, что понятие предела функции в точке вводится только для предельных точек области определения функции. Отметим, что при этом функция может быть и не определена в точке, т.е., вообще говоря, не принадлежит. Число b называется пределом функции в точке, если для любой сходящейся к последовательности! такой, что ∈ , # , соответствующая последовательность значений функции! сходится к b. Обозначение: lim → или → при → . Высшая математика для чайников. Предел функции 2011 год 13 Определения 1 и 2 предела функции эквивалентны. Пусть и O определены в некоторой окрестности точки, кроме, может быть, самой точки, и lim → , lim → OE. Тогда: lim → P O Q E ; lim → P O Q E lim → O E ; lim → O E при условии E 0 Пусть,O и T определены в некоторой окрестности точки, кроме, может быть, самой точки, и удовлетворяют неравенствам (O (T. Пусть lim → lim → T . Тогда lim → O. Здесь, похоже, все понятно. Теоремы выражены четко и ясно, информация должна восприниматься легко. Если что-то не так, не волнуйтесь, примеры нас ждут впереди. 2.Теоремы о пределах Высшая математика для чайников. Предел функции 2011 год 14 Односторонние пределы… Не слишком позитивно звучит, не правда ли? На самом деле все очень просто. На рис. 3 изображён график функции. Давайте попробуем взять пару пределов. Думаю, у нас все получится! 1) Если →1. lim → 1 7 11 естьопределенность 8 1 2) Если →0. lim → неопределенность Следовательно, мы не имеем права дальше решать, а упростить никак нельзя. Следовательно, предела не существует. Посмотрите на рис. 3 и вы увидите, что функция там не определена, сл. Ни о каком пределе не может быть и речи. 3) Если →0 0. Запись →0 0 в данном случае означает “посмотрите на то, как ведет себя функция справа от 0”. И что мы видим на графике? Функция возрастает в + бесконечность. Поэтому: lim → 1 7 1 0 0 определенность 8 ∞ Понимаете? 0 0 0, следовательно, мы уже делим не на ноль. Давайте рассмотрим следующие примеры. 4) Если →0 0. Что у нас делает функция слева от 0? Правильно, убывает. Причем убывает к ∞. lim → 1 7 1 0 0 определенность 8 ∞ Ну как вам? 5) Если →∞ 3.Односторонние пределы Рис. 3 Высшая математика для чайников. Предел функции 2011 год 15 Смотрим на график и видим, что функция при →∞ стремится к 0. lim → 1 7 1 ∞ определенность 8 0 6) Если →∞ Все то же самое: lim → 1 7 1 ∞ определенность 8 0 Последние два примера рекомендую запомнить. При раскрытии неопределенности, они нам потом очень понадобятся. Ну что, поняли суть? Ну, тогда формальности… Определение 1 (предел функции по Коши) Определение 2 (предел функции по Гейне) В общем, добавить тут и нечего. Полная аналогия с предыдущими определениями по Коши и по Гейне, так что, если вы поняли, как доказываются пределы, то сможете доказать и односторонние. Структура доказательств та же. Обозначения: lim → && 0 Если существуют 0 и 0, причем 0 0 , то существует lim → . Число b называется правым (левым) пределом функции в точке a , если для любой сходящейся к a последовательности! такой, что, соответствующая последовательность значений функции! сходится к b. Число b называется правым (левым) пределом функции в точке a , если ∀ 0 ∃ 0 такое, что ∀ , удовлетворяющего условиям, & (, выполняется неравенство | | . Высшая математика для чайников. Предел функции 2011 год 16 Если функция определена в некоторой окрестности точки a, за исключением, быть может, самой точки a, и существует lim → , то существуют 0 и 0, причем 0 0 . На всякий случай, рассмотрим пример на теорему 4. Давайте рассмотрим функцию √ . Она изображена на рис. 4. Давайте найдем пределы: lim → √ V √ 4 0 определенность W 2 Почему 0 ни на что не повлиял? Да потому что ему незачем что-то менять. Функция определена в 4, следовательно, нет никакой надобности брать 0. lim → √ V √ 4 0 определенность W 2 Все то же самое. Функция определена в 4, следовательно, нет никакой надобности брать 0. Этого никто не объясняет, потому что это вполне все логично. Отсюда, по теореме 4: lim → √ ,lim → √ существуют,причемlim → √ lim → √ 2 Поэтому существует предел lim → √ 2. Так, это закрепили. А что, если мы рассмотрим 0? Ну, давайте проверять: lim → √ V √ 0 0 определенность W 0 Этот предел существует. Посмотрите на функцию, и вы увидите, что она там определена. lim → √ V √ 0 0 неопределенность W пределнесуществует Запомните раз и на всегда: корень не может быть отрицательным! Поэтому предела не существует! Но зато существует вот что: lim → √ V √ 0 определенность W 0 Как видите, теорема 4 работает лишь в одну сторону. В ней нельзя поставить отрицание. Поэтому, друзья, будьте внимательны! Рис. 4 Высшая математика для чайников. Предел функции 2011 год 17 Некоторые случаи мы уже рассмотрели (раскрытие неопределенности (часть 1)). От неопределенности мы избавляемся с помощью преобразований! Запомните это, пожалуйста, и ничего не бойтесь. А сейчас я Вам хочу поведать одну небольшую тайну: если →∞, то в большинстве случаев выражение, находящееся под знаком предела, стоит преобразовывать к формам вида E ⁄ , где c – число. Почему? Потому что эта дробь всегда будет стремиться к 0! Мы с вами это уже доказали. Запомните и всегда пользуйтесь этим! №1. Посчитать предел: lim → 5 lim → ]1 5 ^ lim → !1 5 # 1 0 1 Ну как вам? Вывод: когда у нас дробь, то мы выносим → сокращаем →пишем ответ. P.S. В квадратных скобках я не буду теперь писать слово определенность☺ №2. Посчитать предел: lim → 2 lim → 4 4 lim → ] 1 4 4 ^ lim → ! 1 4 4 # 0 0 0 0 Круто? Да! Значит, давайте сделаем и еще одно наблюдение: в таких случаях выносим ту же степень, что и в знаменателе. Хотя, если самая высокая степень стоит в числителе, то лучше вынести именно ее. В общем, как вам удобнее. Можно делать и так, и так. №3. Посчитать предел: lim → 4 2 ∞∞ неопределенность lim → 8 16 4 4 lim → ] 8 16 ^ ]1 4 4 ^ lim → 8 1 4 4 lim → ]1 8 ^ ] 1 1 ^ lim → 1 8 1 7 10 8 ∞ №4. Посчитать предел: lim → " 0 4.Предел функции при (→ ∞ Высшая математика для чайников. Предел функции 2011 год 18 lim → 3 4 1 2 5 2 lim → ]3 4 1 ^ ] 2 5 2^ lim → 3 4 1 2 5 2 7 3 0 0 0 0 2 8 32 №5. Посчитать предел: lim → 2 5 1 6 1 lim → ]2 1 5 1 ^ ]6 1 1 ^ lim → 2 1 5 1 6 1 1 lim → 2 1 6 ∞ №6. Посчитать предел: lim → 1 2 4 4 lim → ] 1 2 4 ^ ] 4 1^ lim → 1 2 4 4 1 7 0 0 0 0 1 8 0 Еще раз повторяю, когда дробь – тогда выносим! Настало время поведать вам и вторую тайну. Если нам дано выражение вида _ `_ , не поленитесь его помножить на. Привожу пример: lim → ∞∞неопределенность lim → ∙ lim → 2 lim → 2 1 ]1 1 ^ lim → 2 1 1 1 lim → ]1 2 1 ^ ] 1 1 ^ 7 10 8 ∞ Несомненно, в будущем вы так не будете все подробно расписывать. Вам будет достаточно нескольких действий, так что не волнуйтесь. P.S. Как только встречаете №1. Посчитать предел: lim → b 8 3 b Сложно? Нет! На какой вид похоже? На _ `_ . Делаем сопряженное. & & С О П Р Я Ж Е Н Н О Е Высшая математика для чайников. Предел функции 2011 год 19 lim → b +8 +3 − b + = lim → P√ +8 +3 − √ + QP√ +8 +3 + √ + Q √ +8 +3 − √ + = lim → +8 +3 − − √ +8 +3 − √ + = lim → 7 +3 c 1 + 8 + 3 + c 1 + 1 = lim → ]7 + 3 ^ d c 1 + 8 + 3 + c 1 + 1 e = lim → 7 + 3 c 1 + 8 + 3 + c 1 + 1 Вот то, что я вам говорил. Вы ВСЕ должны в конечном итоге получать дроби вида с, потому что все они стремятся к 0!!! Продолжаем: lim → 7 + 3 c 1 + 8 + 3 + c 1 + 1 = 7 7 +0 √ 1 +0 +0 + √ 1 +0 8 = 72 Страшно? Ну нет же☺. Медленно, не спеша, решайте пределы и вы достигните многого! №2. Посчитать предел: lim → c + b + √ √ +1 Страшно☺? Не волнуйтесь, все то же самое. Надо что-то сократить. Что и как? √ −это надо вынести и сократить. Если попытаемся вынести, то мы с вами просто запутаемся, а ответ от этого не изменится. Разве что может быть неопределенность. То есть выносим x с самой старшей степенью в знаменателе. lim → c + b + √ √ +1 = lim → √ ∙ f 1 + g 1 + c 1 √ ∙ c 1 + 1 = lim → f 1 + g 1 + c 1 c 1 + 1 = h i i i i i i j f 1 + g 10 + c 1 0 c 1 + 10 k l l l l l l m = 1 Трудность может состоять здесь лишь в одном: как вынести √ ? Надеюсь, что это вы делать умеете. №3. Посчитать предел: lim → P −√ −1 Q + P +√ −1 Q Высшая математика для чайников. Предел функции 2011 год 20 Кем бы ни был наш чужак, мы все равно его решим. Для начала давайте, используя теорему 2, разобьем наш предел на два предела. Его так будет намного легче решать, в том смысле, что можно меньше запутаться. Если боитесь разбивать, то сами мучайтесь. ☺ lim → P −√ −1 Q + P +√ −1 Q = lim → P −√ −1 Q + lim → P +√ −1 Q = lim → d −√ −1 e + lim → d +√ −1 e Мы просто все упростили для дальнейшей работы с пределами, используя сложение дробей и свойство степени. Теперь у нас два предела. Видим дробь. Как я вас учил? Правильно, видим дробь – умножаем на сопряженное. Так давайте сделаем это вместе. lim → d −√ −1 e + lim → d +√ −1 e = lim → d P −√ −1 QP +√ −1 Q ∙ P +√ −1 Q e + lim → d P +√ −1 QP −√ −1 Q ∙ P −√ −1 Q e Вот, что у нас получилось. Заметьте, делаем то же самое, что и раньше. Отличие лишь в одном – размеры. Теперь надо упростить каждый предел. В числителе у нас разность квадратов. Упростим первый предел: lim → d P −√ −1 QP +√ −1 Q ∙ P +√ −1 Q e = lim → n − P √ −1 Q ∙ P +√ −1 Q o = lim → d − +1 ∙ P +√ −1 Q e = lim → d 1 ∙ P +√ −1 Q e Первый упростили. Теперь перейдем ко второму: lim → d P +√ −1 QP −√ −1 Q ∙ P −√ −1 Q e = lim → d 1 ∙ P −√ −1 Q e Вот, что у нас получилось: lim → P −√ −1 Q + P +√ −1 Q = lim → d 1 ∙ P +√ −1 Q e + lim → d 1 ∙ P −√ −1 Q e Видим дробь. Что надо делать? ВЫНОСИТЬ! Первый предел: Высшая математика для чайников. Предел функции 2011 год 21 lim → d 1 ∙ P +√ −1 Q e = lim → ! 1 + ∙ √ −1 # = lim → p q r ∙ 1 d 1 + c 1 − 1 e s t u = 7 02 8 = 0 Второй предел: lim → d 1 ∙ P −√ −1 Q e = lim → ! 1 − ∙ √ −1 # = lim → p q r ∙ 1 d 1 − c 1 − 1 e s t u = 7 00 −неопределенность! 8 Друзья, вот с таким вот вы будете сталкиваться часто, особенно на больших примерах. Что делать? Ответ прост: вернуться и сделать по-другому. Хорошо, что хотя бы первый предел у нас посчитался. Что же, возвращаемся до разбиения лимитов. Вот что у нас было: lim → d +√ −1 e Как решать, если наш способ не подошел? Что делать, если “метод сопряженных” не работает. А давайте сразу попробуем вынести? Выносим со старшей степенью в знаменателе, следовательно это просто. lim → d +√ −1 e = lim → p q r d 1 + c 1 − 1 e s t u = lim → n1 + g 1 − 1 o = V 1 + √ 1 −0 W = 2 Получается, на самом деле, все было несколько проще. Итого: lim → P −√ −1 Q + P +√ −1 Q = lim → d −√ −1 e + lim → d +√ −1 e = 0 +2 Все! Ответ: 2 Сложно? Я думаю, не очень. Здесь главное аккуратность и настойчивость. Если сразу не получилось, не надо все бросать. №4. Посчитать предел: Высшая математика для чайников. Предел функции 2011 год 22 lim → √ 4 − − √ 4 + 3 Здесь у нас не стремится к бесконечности, но я хочу тем показать, что метод сопряженного действует и здесь. lim → √ 4 − − √ 4 + 3 = lim → P √ 4 − − √ 4 + QP √ 4 − + √ 4 + Q 3 P √ 4 − + √ 4 + Q = lim → 4 − −4 − 3 P √ 4 − + √ 4 + Q = lim → −2 3 P √ 4 − + √ 4 + Q = − 23 lim → 1 √ 4 − + √ 4 + = − 16 №5. Посчитать предел: lim → √ +1 −1 √ +2 − √ 2 Здесь сделаем еще круче – умножим числитель и знаменатель на выражения сопряженные числителю и знаменателю. lim → √ +1 −1 √ +2 − √ 2 = lim → P√ +1 −1 QP√ +1 +1 QP√ +2 + √ 2 Q P√ +2 − √ 2 QP√ +1 +1 QP√ +2 + √ 2 Q = lim → (+1 −1) P√ +2 + √ 2 Q (+2 −2) P√ +1 +1 Q = lim → P√ +2 + √ 2 Q P√ +1 +1 Q = lim → √ +2 + √ 2 √ +1 +1 = √ 2 №6. Посчитать предел: lim → b 1 +tg − b 1 −tg sin2 = lim → P b 1 +tg − b 1 −tg QP b 1 +tg + b 1 −tg Q sin2 P b 1 +tg + b 1 −tg Q = lim → 2tg sin2 P b 1 +tg + b 1 −tg Q = lim → 1 cos P b 1 +tg + b 1 −tg Q = 12 Высшая математика для чайников. Предел функции 2011 год 23 Итак, какой вывод мы можем сделать из всего предыдущего? Ну, во-первых, если вас просят посчитать предел, то уж наверняка, там – неопределенность. Таблички снизу рекомендую вам заучить!!! Пример: lim → lim → lim → 2 lim → ]1 2 1 ^ ] 1 1 ^ lim → 1 2 1 1 1 7 1 0 0 0 0 8 ∞ & & , Р А С К Р Ы Т И Е Н Е О П Р Е Д Е Л Е Н Н О С Т И 2) Если у нас есть выражение типа, и в итоге получается неопределенность, то нам нужно провести вот такую операцию: а потом вынести и сократить так, что бы во всех случаях был в знаменателе. , Р А С К Р Ы Т И Е Н Е О П Р Е Д Е Л Е Н Н О С Т И 1) Если у нас есть выражение типа, и в итоге получается неопределенность, то нам нужно провести вот такую операцию: а потом вынести и сократить так, что бы во всех случаях был в знаменателе. Высшая математика для чайников. Предел функции 2011 год 24 Пример: lim → lim → lim → lim → ]1 1 ^ ] 1 1 ^ lim → 1 1 1 1 7 1 0 0 0 8 ∞ Как видите, мы один и тот же предел посчитали разными способами. Такое получается не всегда! Все таблицы Вы должны запомнить, как таблицу умножения. Наверное, у многих может возникнуть вопрос: а когда что использовать? Практика, друзья. Другого выхода у Вас нет, и не может быть. Только на собственном опыте Вы можете достигнуть каких-то результатов. Как всегда, переходим к формальностям (профессорской теории):) * "*+ , Р А С К Р Ы Т И Е Н Е О П Р Е Д Е Л Е Н Н О С Т И 3) Если у нас есть выражение типа То вам нужно либо сразу выносить и сокращать так, что бы во всех случаях был в знаменателе, либо помножать на сопряженное числителя или знаменателя. В зависимости от ситуации. Все три выше приведенные пункты вы должны использовать при раскрытии неопределенности, когда → ∞ . Если стремится к какому-то другому значению, и у нас неопределенность, то используют просто упрощения (сопряженное или сокращения) Пусть функция определена на прямой " , & ∞ . Число называется пределом функции при → & ∞ lim → , если ∀ 0 ∃ , 0 - " такое, что ∀ , выполняется неравенство | | . Высшая математика для чайников. Предел функции 2011 год 25 Пусть функция определена на прямой " , & ∞ . Число называется пределом функции при → & ∞ , если для любой бесконечно большое последовательности! " соответствующая последовательность значений функции! сходится к. Высшая математика для чайников. Предел функции 2011 год 26 То же самое и для бесконечно малых функций. На мой взгляд, определение нам нужно либо для доказательств, либо… для других целей. По крайней мере, оно мне ни разу не понадобилось. Итак, мы с вами уже встречали ранее примеры, когда предел был равен ∞. Как видите, они считаются точно также, как и все другие. Ключевую роль здесь играет вот такая конструкция: V 1 0 v W . Запомните, эта конструкция ВСЕГДА равна ∞! | | . . Функция называется бесконечно большой в точке a справа, если ∀ . 0 ∃ 0 такое, что ∀ , удовлетворяющего условию, &, выполняется неравенство Обозначение: lim → ∞ Функция называется бесконечно большой при → & ∞ , если ∀ . 0 ∃ , - " такое, что ∀ , | | . . Обозначение: lim → ∞ 5.Бесконечно большие функции 0 1 0 1 2 ∞ Высшая математика для чайников. Предел функции 2011 год 27 Да, именно это нам сейчас и предстоит. Они нам ОЧЕНЬ понадобятся в будущем. Поэтому, важно их сейчас же закрепить, а заодно и посчитать пределы. Я согласен, это нудно и неинтересно. Если Вы что-то знаете, пропускайте и идите дальше, я разрешаю☺. Итак, это наша первая и самая важная функция. Ранее мы уже успели ее рассмотреть, но давайте повторим то, что уже сделали. lim → w ∞ lim → w 0 lim → w ∞ lim → w 0 Если хотите, можете запомнить все это, но вообще, я рекомендую вам запомнить сам график. По- моему все довольно ясно. Ну, эту функцию вы просто обязаны знать, но, на всякий случай я ее напомню. Знаете ли, разные случаи бывают☺. lim → ∞ lim → ∞ 6.Графики элементарных функций 3 1 & & " Высшая математика для чайников. Предел функции 2011 год 28 Функция носит свое название – показательная функция. Здесь важно не забывать об одной вещи: при 1 функция возрастает; при 0,1 функция убывает. Здесь давайте рассмотрим примеры: №1. Посчитать предел 1 lim → 2 2 ∞ lim → 2 2 0 ЗАЗУБРИТЬ! Вот это вы просто обязаны заучить, потому что графики часто путают между собой. №2. Посчитать предел 0,1 lim → ! 12 # lim → 1 2 7 1 2 1 ∞ 8 0 lim → ! 12 # lim → 1 2 7 1 2 10 8 ∞ Как видите, последние два предела мы просто вывели из предыдущих двух. ЗАЗУБРИТЬ! Функция носит свое название – логарифмическая функция. Здесь есть тоже два подвоха: при 1 функция возрастает; при 0,1 функция убывает. №1. Посчитать пределы 1 lim → log 0 lim → log ∞ lim → log ∄ lim → log ∄ №2. Посчитать пределы 0,1 log Высшая математика для чайников. Предел функции 2011 год 29 lim → log 0 lim → log ∞ lim → log ∄ lim → log ∄ Уверен, столько всего вы не запомните, так что лучше выучить график. Ok! Идем дальше… Функция носит свое название – синусоида. №1. Посчитать предел lim → sin. Что делать? На графике явно видно, что функция “прыгает” от одного значения до другого. Вывод: не существует такого предела. Давайте просто рассмотрим примеры, где функция стремится к разным значениям: lim → sin { | } | ~ lim → sin1 lim → sin 0 lim → sin 1 ; Проделам то же самое для косинусоиды. №1. Посчитать предел: lim → cos. Все те же размышления. Предела не существует! Вот, что у нас получается: lim → cos { | } | ~ lim → cos0 lim → cos 1 lim → cos 1 ; sin "67 Высшая математика для чайников. Предел функции 2011 год 30 На рисунке представлены две функции: O и EO. Как видите, они очень похожи, поэтому очень важно, запомните Вы их или нет. Давайте проведем небольшой опыт. Попробуйте запомнить два графика. Как только будете уверены в том, что все выучили, прорешайте все пределы ниже, а потом проверьте себя по графикам. №1. Посчитать пределы: lim → tg lim → tg lim → tg lim → tg lim → tg lim → tg lim → ctg lim → ctg lim → ctg lim → ctg lim → ctg lim → ctg arcsin – обратная функция к функции sin. arccos – обратная функция к функции cos. №1. Посчитать предел: lim → arcsin. Давайте посмотрим на график arcsin. Что мы видим? При → 0 функция принимает бесконечно много значений. Например, lim → arcsin0 и lim → arcsin и т.д. Делаем вывод: у нашего графика есть период. lim → arcsinw,w целоечисло,лежащеевпромежутке∞,∞ 89 "89 arcsin arccos Высшая математика для чайников. Предел функции 2011 год 31 То же самое с arccos. arctg – обратная функция к функции tg. arcctg – обратная функция к функции ctg. №1. Посчитать предел: lim → arctgw ∙ 2 w целое число, имеющее шаг 2. Т.е. lim → arctg ⋯. Можно записать вот так: lim → arctg 2 2 2 w Заметим, что это произвольное целое число, которое мы задаем сами. На этом, мы заканчиваем наш раздел – графики элементарных функций. От автора: Поздравляю! Вы смогли завершить первую главу “Предел функции” первой части “Предел и непрерывность функции”. Конечно, это не все. Я рассказал Вам лишь элементарные вещи. Далее нас будут ждать первый замечательный и второй замечательный приделы и другие методы взятия пределов. Если Вы поняли все, что я здесь написал, то дальше будет только интересно! Ничего сверхсложного вас не ожидает… arctg arcctg Высшая математика для чайников. Предел функции 2011 год 32 Глава 2. Непрерывность функции в точке. Запомните это определение раз и навсегда! Если вы его не знаете, вы – ничто и никто в математике. Давайте рассмотрим простой пример: 1 Задание: проверить функцию на непрерывность в точках 1;0. 1. 1. Используя определение 1, получаем: lim → 1 1 ↭ 1 11 1 Выполняется определение 1? Да! lim → 1 1 1 Вывод: функция непрерывна в точке 1. 2. 0. Используя определение 1, получаем: lim → 1 ∞↭ 0 10 →∄ Выполняется определение 1? Нет! lim → 1 0 lim → Функция называется непрерывной в точке a, если 1.Непрерывность функции в точке. Содержание: 1) Непрерывность функции в точке 2) Непрерывность сложной функции 3) Классификация точек разрыва 4) Непрерывность элементарных функций 5) Первый замечательный предел 6) Второй замечательный предел 7) Кратко о Maple Высшая математика для чайников. Предел функции 2011 год 33 Вывод: функция не существует в точке 0. Здесь то же самое. Пожалуйста, рассмотрите сами такие функции как ln, и другие. Хотя, думаю, что все предельно ясно. Для того чтобы функция была непрерывна в, необходимо и достаточно, чтобы она была непрерывна в этой точке справа и слева. Если функции и O непрерывны в точке, то функции O, O, O, /O также непрерывны в точке (частное – при условии O 0). Пример №1. Исследовать на непрерывность функцию. Для начала распишем область определения D∞,0 ∪0,∞, т.к. знаменатель не может равняться 0. Теперь просто используем теорему 6: lim → , где 0. Следовательно, по теореме 6, функция непрерывна в любой точке, кроме 0. lim → > соответственно lim → E . Пусть функция определена в правой (левой) полу окрестности точки a, т.е. на некотором полуинтервале, & (соответственно,). Функция называется непрерывной справа (соответственно слева) в точке a, если Высшая математика для чайников. Предел функции 2011 год 34 Впрочем, пока что вам это не сильно понадобиться. Привожу примеры сложных функций: b | sin | ,cos 1 ,log 1 . Почему они сложные? Давайте рассмотрим цепочку последовательных преобразований для первой из них: sin | | √ . Вот и все! Теперь перейдем ко второй функции: 1 cos . И так далее. Не хочется уделять этому много времени. Надеюсь, вы и так все поняли. Ну что же, перейдем к теореме. Пусть функция непрерывна в точке, а функция непрерывна в точке. Тогда сложная функция P Q непрерывна в точке. Давайте рассмотрим пример на доказательства. Здесь как раз и нужно рассматривать сложную функцию. Пример №1 Доказать, что: lim → 1 ln, 0, 1. Рассмотрим функцию 1. Она непрерывна в точке 0 и 0 0. При этом Пусть функция F определена на множестве, а G множество значений этой функции. Пусть, далее, на множестве G определена функция H . Тогда говорят, что на множестве определена сложная функция, и пишут H , где F , или H F . 2.Непрерывность сложной функции. Высшая математика для чайников. Предел функции 2011 год 35 log 1 , 1 log 1 . Вычислим lim → : lim → log 1 lim → ln ln 1 Этот шаг может быть непонятен, поэтому я должен напомнить вам формулу преобразования к логарифму с другим основанием: Запомните ее и больше не возвращайтесь к этому. В данном случае новое основание. Давайте напишем формулу именно для нашего случая: log 1 log 1 log ln1 ln . Итак, продолжаем: lim → log 1 lim → ln ln 1 ln 1 lim → ln1 . Верно? ln это число, поэтому мы его и вынесли. Теперь нужно посчитать предел lim → . Представим функцию в виде ln 1 ln (тоже свойство логарифма!), где 1 . Так как lim → 1 (Это второй замечательный предел. Пока что мы его не прошли, но, поверьте, равенство верно), а функция ln непрерывна в точке, то lim → ln 1 ln1. Возвращаемся к нашему примеру. И вот, что у нас получается: log log log ∙ log log Высшая математика для чайников. Предел функции 2011 год 36 lim → log (1 +) = lim → ln ln 1 + = ln 1 lim → ln(1 +) = ln 1 = ln. Рассмотрим теперь функцию (), непрерывную в точке = 0: = log (1 +) при ≠ 0 lnпри = 0 Согласно теореме 8 сложная функция P Q = −1 при ≠ 0 lnпри = 0 Является непрерывной в точке = 0. Поэтому lim → −1 = ln. Сложно? Может быть, но вы должны в этом разобраться, потому что это очень важно для понимания этой темы. Тем более, здесь требуется внимательность, ну и “немножко подумать”. Высшая математика для чайников. Предел функции 2011 год 37 Для начала, давайте поймем, что вообще означает “точка разрыва”. Все предельно просто! Прежде чем начинать рассматривать классификацию точек разрыва, вы должны всегда проверять условие: должна быть определена в некоторой окрестности точки, за исключением, быть может, самой точки. Если условие выполняется, то можно рассматривать классификацию точек разрыва. Пример №1. sin Прежде всего, напишем область определения: D ∞;0 ∪0;∞. Отсюда сразу видно, что 0 необычная точка. В ней функция не определена, но определена в ее окрестности. lim → sin 1 0 sin . Отсюда следует, что 0 устранимая точка разрыва. Точка называется точкой разрыва функции, если в этой точке не является непрерывной. lim → # Точка – устранимая точка разрыва, если 3.Классификация точек разрыва. Высшая математика для чайников. Предел функции 2011 год 38 Пример №1. sgn Функция sgn уже должна быть ранее вам известна, но я вам ее напомню. sgn 1,0, 1, 0 0 ,0 , lim → sgn 1, lim → sgn 1, 0 0. Отсюда следует, что lim → sgn lim → sgn sgn точка 0 точка разрыва первого рода. Пример №1. tg Прежде всего, напишем область определения D \ 2 w ,w0. lim → tg∞ ∃ lim → # lim → # Точка – точка разрыва первого рода, если Точка – точка разрыва второго рода, если хотя бы один из односторонних пределов не существует или равен бесконечности. f(x) = sgn(x) Высшая математика для чайников. Предел функции 2011 год 39 lim → tg∞ Т.к. хотя бы один из пределов равен бесконечности, то w точка разрыва второго рода. Пример №2. ln Прежде всего, напишем область определения D 0;∞. limln → 0 limln → ∄ Т.к. хотя бы один из пределов не существует, то 0 точка разрыва второго рода. Итак, мы теперь знаем классификацию точек разрыва. Мы рассмотрели примеры к каждому случаю. Они достаточно легкие, поэтому давайте еще попрактикуемся. Во всех следующих номерах определить точки разрыва. P.S. Для начала попробуйте сделать это сами, ну а потом проверьте себя. Удачи ☺! №1. 2 , ln, (1 1 lim → lim → ln0, lim → lim → 1. lim → lim → В т. 1 функция имеет разрыв первого рода. №2. Прежде всего, напишем: D ∞,0 ∪0,∞. Высшая математика для чайников. Предел функции 2011 год 40 lim → lim → 7 0 8 0, lim → lim → ∄. 0 предельная точка второго рода. №3. 1 2 3 Прежде всего, напишем: 4 0 D ∞,4 ∪4,∞. lim → 1 2 3 lim → 1 2 3 7 1 2 0 8 12 , lim → 1 2 3 lim → 1 2 3 7 1 ∞ 8 0. 4 точка разрыва первого рода. №4. | 1 | Прежде всего, напишем. Критические точки определяем вот так: 0 1 0. Критические точки: 0 и 1. Теперь напишем область определения D ∞,0 ∪ 0,1 ∪1,∞. lim → | 1 | 7 10 8 ∞ 0 точка разрыва второго рода. lim → | 1 | lim → 1 lim → 1 1 lim → 1 1 Высшая математика для чайников. Предел функции 2011 год 41 lim → | 1 | lim → 1 lim → 1 1 lim → 1 1 1 точка разрыва первого рода. 0 точка разрыва второго рода, 1 точка разрыва первого рода. №5. 1 1 Прежде всего, напишем: D ∞,1 ∪1,∞. lim → 1 1 lim → 1 1 1 lim → 1 1 13 Точка разрыва устранимая: F 1 1 , 1 13 ,1 Она непрерывна в точке разрыва и на D. №6. 1 1 1 1 1 1 Что бы найти критические точки, нужно упростить функцию. 1 1 1 1 1 1 1 1 Точки: 0;1;1. lim → 1 устранимыйразрыв. lim → ∞разрыввторогорода. lim → 0 устранимыйразрыв. Высшая математика для чайников. Предел функции 2011 год 42 №7. cos cos 1 и получаем: 2 2w 1 устранимыеточкиразрыва. 0 точкаразрывавторогорода. Думаю, примеров достаточно. Если вы сами все это про решаете, то тему вы знать будете на 100%. Ну что же, надеюсь, это было не слишком скучно. По крайней мере, столько разобранных примеров вы не найдете нигде. Высшая математика для чайников. Предел функции 2011 год 43 Мы с вами эту тему уже разобрали в 1 главе, 6 пункте. Там мы рассматривали графики элементарных функций и считали пределы. Сейчас перейдем к формальностям и “профессорской теории”. Как вы заметили, в моей книге присутствует эта “теория”. Зачем? Все просто, - хочется, что бы вы не только принимали разжеванное, но и сами пытались разжевать. Если я уберу эту “теорию”, то мои труды пойдут насмарку. Конечно, вы будете уметь что-то решать, но вы не будете понимать, что да как. Поэтому прошу вас учить теорию! Она обязательно понадобиться вам в ближайшем будущем. Ну что же, это было лирическое отступление ☺. Перейдем к небольшой теории. Любая элементарная функция, определенная в окрестности некоторой точки, непрерывна в этой точке. На этом “профессорская теория” заканчивается, и мы переходим к замечательным пределам. Функции I "6J78 , log 0 , # 1 , sin , cos , tg , ctg , arcsin , arccos , arctg , arcctg называются простейшими (или основными) элементарными функциями. Совокупность всех элементарных функций называется классом элементарных функций. Функция называется элементарной, если она может быть получена с помощью конечного числа арифметических операций и суперпозиций над простейшими элементарными функциями. 4.Непрерывность элементарных функций Высшая математика для чайников. Предел функции 2011 год 44 Очень важная тема! В ней мы будем учиться искать пределы. Вы должны набить руку на этом, и у меня к вам просьба: перед тем, как смотреть решение, попытайтесь сами чего-то добиться. Зазубрите это раз и навсегда! И никогда не забывайте эту формулу! Доказывать я ее не собираюсь, если хотите, поищите в интернете, там она точно есть. Ну что же, переходим к примерам. №1. lim → sin . Решение: sin 1 sin , Ура! Внизу появился замечательный предел. lim → sin lim → 1 sin 7 11 8 1. Легко? Безусловно… №2. lim → arcsin . Решение: Сделаем замену переменной: пусть arcsin. Тогда sin и база →0 переходит в базу →0 (просто подставьте →0 под arcsin). На самом деле это проще записывать вот так: lim → arcsin 7 arcsin ↭sin → 0 ↭ →0 8 lim → sin 7 11 8 1. 5.Первый замечательный предел lim → sin 1 Высшая математика для чайников. Предел функции 2011 год 45 Запомните этот способ замены переменной. Он может сильно пригодиться вам в будущем. №3. lim → arcsin . Решение: lim → arcsin lim → 1 arcsin 7 arcsin ↭sin →0 ↭ →0 8 1 lim → sin 7 11 8 1. №4. lim → sin2 sin3 . Решение: Преобразуем функцию следующим образом: lim → sin2 sin3 lim → ! sin2 2 ∙ 3 sin3 ∙ 23 #. Вынесем постоянный множитель за знак придела и применим теорему о пределе произведений: lim → sin2 sin3 lim → ! sin2 2 ∙ 3 sin3 ∙ 23 # 23 ∙ lim → sin2 2 ∙ lim → 3 sin3 Делаем замену, как и в предыдущем примере: lim → sin2 sin3 lim → ! sin2 2 ∙ 3 sin3 ∙ 23 # 23 ∙ lim → sin2 2 ∙ lim → 3 sin3 ! 2 ↭sin2sin →0 ↭ →0 4 3 ↭sin3sin →0 ↭ → 0 # 23 ∙ lim !→ sin ∙ lim "→ 1 sin 23 ∙ 1 ∙ 1 23 . №5. lim → sin 4 . Умножим и разделим знаменатель на 4 и подведем выражение под знаком предела к первому замечательному пределу. lim → sin 4 lim → sin 4 4 ∙ 4 14 ∙ lim → sin 4 4 d 4 ↭4 →0 ↭ →0 e 14 ∙ lim !→ sin 14 . Высшая математика для чайников. Предел функции 2011 год 46 №6. lim → 2tg 2 . Представим тангенс через синус и косинус и воспользуемся теоремами о пределах. lim → 2tg 2 lim → 2 ∙ sin 2 cos 2 lim → 2sin 2 cos 2 2 lim → sin 2 4] 2 ^ ∙ lim → 1 cos 2 d 2 ↭2 → 0 ↭ → 0 e 12 lim → sin ∙ lim → 1 cos 2 7 12 ∙ 1 ∙ 11 8 1. Видите, здесь немного посложнее, но в принципе, все одно и тоже. Если вы выучили элементарные функции, то это вам не должно показаться сложным. №7. lim → 1 cos 2 tg . По формулам двойных углов имеем: lim → 1 cos 2 tg lim → 1 cos sin tg lim → cos sin cos sin tg lim → cos sin cos sin tg lim → 2sin tg lim → 2sin cos sin cos 2 lim → sin lim → cos 2 ∙ 1 ∙ 1 2. Господа, учим тригонометрические формулы! Они вам все равно понадобятся. Высшая математика для чайников. Предел функции 2011 год 47 Формул много, но желательно их все выучить. №8. lim → 8sin 4 . Умножим и разделим числитель на 4 в кубе: sin K *L sinKcos L *cosKsinL cos K *L cos Kcos L ∓sinKsinL t9 K *L t9K *t9L 1 ∓t9Kt9L ct 9 K * L ct 9 K" t 9L ∓ 1 ct 9L * ct 9 K sin K &cos K 1 tg K &1 1 cos K ctg K &1 1 sin K sin2K 2sinKcos K cos 2K cos K sin K 2cos K 1 1 2sin K tg 2K 2tgK 1 tg K ctg2K ctg K 1 2ctgK sinK *sinL 2sin K *L 2 cos K ∓L 2 cos K &cosL 2cos K &L 2 cos K L 2 cos K cos L 2 sin K & L 2 sin K L 2 Высшая математика для чайников. Предел функции 2011 год 48 lim → 8sin 4 lim → 4 ] 4 ^ 8sin 4 8 lim → ] 4 ^ sin 4 d 4 ↭4 →0 ↭ →0 e 8lim !→ sin 8 ∙ 18. №9. lim → sin 2 4 1 . В знаменателе мы можем сделать квадрат разности, а потом, как всегда, перейти к новой переменной. Тогда предел будет стремится к 0, и, следовательно, мы можем применить первый замечательный предел. lim → sin 2 4 1 lim → sin 2 2 ] 2 ↭ 2 →2 ↭ →2 20 ^lim !→ sin 1 1. №10. lim → sin3 sin4 6 . На основании одной из теорем о пределах, мы можем данный предел разделить на два предела: lim → sin3 sin4 6 lim → sin3 6 lim → sin4 6 12 lim → sin3 3 23 lim → sin4 4 3 ↭ 3 →0 ↭ →0 4 ↭ 4 → 0 ↭ →0 ¡ 12 lim → sin 23 lim → sin 76 . №11. lim → cos cos 3 . Преобразуем числитель с помощью формул разности косинусов двух углов и синуса двойного угла: cos cos32sin2 sin4sin cos , тогда lim → cos cos3 4lim → sin cos 4lim → cos4. Высшая математика для чайников. Предел функции 2011 год 49 Вторым замечательным пределом называется предел вида Доказывать это мы с вами тоже не собираемся. Быть может, когда-нибудь я напишу отдельно книгу про все доказательства, но пока что не будем тратить на это время и сразу перейдем к примерам. Как только вы видите скобку в степени, значит прежде всего пробуйте ее свести ко второму пределу. Первые номера рассмотрим крайне подробно. №1. Посчитать предел: lim → ! 4 # Видим скобку в степени 5, следовательно пробуем свести к второму замечательному пределу. Сначала сведем то, что внутри к форме 1: lim → ! 4 # lim → !1 4 # Теперь нужно “поиграть” со степенью. Т.е. нам нужен вид типа /4. Почему? Формулу lim → !1 1 # можно было бы представить в виде lim → !1 1 # . В данном случае у нас вместо единицы – четверка. Значит, вот, что у нас получается: lim → ! 4 # lim → !1 4 # lim → ¢ !1 4 # £ . Что бы уж полностью свести к нашей формуле данный придел, мы обозначим 4. Тогда получаем: lim → 1 1 lim → 1 6.Второй замечательный предел Высшая математика для чайников. Предел функции 2011 год 50 lim → ! +4 # = lim → !1 + 4 # = lim → ¢ !1 + 4 # £ = ¤ = 4 ↭ = 4 →∞↭ → ∞ ¥ = lim !→ ¦!1 + 1 # ! § = . Как видите, ничего сложного здесь нет. Алгоритм работы весьма просто: приведение дроби к виду 1 + # приведение степени к виду # ∙ ¨ замена переменной а далее просто считаем по формуле. Если запутались, не волнуйтесь. Мы еще успеем разобрать массу примеров ☺. №2. Найти предел: lim → ! +2 +1 # Действуем так же, как и в прошлый раз: lim → ! +2 +1 # = lim → ! +1 +1 +1 # = lim → !1 + 1 +1 # Здесь мы степень выделять будем после замены переменной. В данном случае, это проще, чем попытаться свести к второму пределу до замены. На результат это никак не повлияет. lim → ! +2 +1 # = lim → ! +1 +1 +1 # = lim → !1 + 1 +1 # = 2 = −1 ↭ = +1 →∞↭ →∞ = lim !→ !1 + 1 # ! = lim !→ ¦!1 + 1 # ! § lim !→ !1 + 1 # = ∙ 1 = . Как видите, ничего сверхъестественного здесь нет. Отсюда можно написать алгоритм решения, подобный прошлому. Приведение дроби к виду 1 + # замена переменной приведение степени к виду # ∙ ¨ а далее просто считаем по формуле. №3. Найти предел: lim → d +5 +2 e Выделим целую часть в скобках: lim → d +5 +2 e = lim → d +2 +3 +2 e = lim → !1 + 3 +2 # = +2 = 3 ↭ = +2 3 →∞↭ →∞ = lim !→ !1 + 1 # ! = lim !→ ¦!1 + 1 # ! § lim !→ !1 + 1 # = ∙ 7 1 + 10 8 = . Высшая математика для чайников. Предел функции 2011 год 51 Пример полностью аналогичен предыдущему. Если вы поняли, как “это работает”, то вы молодцы и можете смело идти дальше. Большой плюс здесь заключается в том, что достаточно знать лишь несколько методов, что бы решить тот или иной предел. №4. Посчитать предел: lim → ! 1 2 # Выделим целую часть в скобках: lim → ! 1 2 # lim → 7 !1 1 # 12 8 lim → !1 1 # lim → ! 12 # 1 ↭ 1 → ∞↭ →0 lim !→ 7 1 ! 8 lim → 2 2 lim → 2 8 ∞ Далее не хочется так подробно рассматривать каждый пример, иначе каждое решение будет занимать более половины страницы. Главное, чтобы вы поняли общую идею, и стремились к идеальному решению, т.е. короткому. Дам еще один совет, попробуйте для начала сами что-то решить, а потом уже проверяйте, верно ли вы сделал ли или нет. №5. Посчитать предел: lim → !1 1 # Решение: lim → !1 1 # 1 ª« ªª lim → !1 1 # ∙!1 1 # ∙ lim → !1 1 # ∙ 1 №6. Посчитать предел: lim → 1 Решение: lim → 1 1 ª« ªª lim → ! 1 # №7. Посчитать предел: lim → !1 2 # Высшая математика для чайников. Предел функции 2011 год 52 Решение: lim → !1 2 # 1 ª« ªª lim → n!1 2 # o №8. Посчитать предел: lim → !1 4 # Решение: lim → !1 4 # 1 ª« ªª lim → n!1 4 # o №9. Посчитать предел: lim → ! 3 1 # Решение: lim → ! 3 1 # 1 ª« ªª lim → ! 1 4 1 # lim → !1 4 1 # ∙ lim → №10. Посчитать предел: lim → 4 ln 2 3 ln5 3 Решение: lim → 4 ln 2 3 ln5 3 lim → 4 ln 2 3 5 3 lim → ln! 2 3 5 3 # lim → ln!1 3 5 3 # % lim → ln1. №11. Посчитать пределы: Высшая математика для чайников. Предел функции 2011 год 53 lim → d +1 +3 e Должен сказать, пример этот уже чуть интереснее предыдущих. Решение: lim → d +1 +3 e = lim → d 1 + +1 +3 −1 e = lim → d 1 + +1 − −3 +3 e = lim → !1 + −2 +3 # = lim → !1 + −2 +3 # ∙ ∙ = lim → ¢ !1 + −2 +3 # £ = &" → = = 1 На этом я предлагаю закончить второй замечательный предел. Далее, в конце книги вы сможете найти массу заданий на эту тему. Разумеется, ответы будут прилагаться. Высшая математика для чайников. Предел функции 2011 год 54 Так же хотелось бы сделать заметку по поводу электронного вычисления пределов. Есть такая программа – Maple, и там пределы считаются просто на «ура». Как видите, слева, в окошке есть шаблоны формул. Просто на них нажимаете и заполняете данными. Нажимаете на Enter и получаете ответ. На скриншоте для примера посчитан наш последний предел. Зачем нужна вам эта программа? Для проверок. Посчитали предел на бумаге, получили ответ. Вбили формулу в программе и проверили. На самом деле очень удобная штука. От автора: Поздравляю! Вы смогли завершить вторую главу “Непрерывность функции в точке” первой части “Предел и непрерывность функции”. Впереди вас ждет сравнение бесконечно малых функций, символ “Ο малое” и его свойства, вычисление пределов функций с помощью асимптотических формул и вычисление пределов показательно-степенных функций. Темы будут весьма важны, поэтому будут рассматриваться не только “технические” примеры, но так же примеры и на доказательства. На этой ноте я хочу вам пожелать успехов! До скорой встречи! Искренне Ваш, Виосагмир И.А. 7.Кратко о Maple Высшая математика для чайников. Предел функции 2011 год 55 Глава 3. Бесконечно малые функции. Функция - называется бесконечно малой при → (в точке), если lim → -0. Пусть - и ® две бесконечно малые функции при →. Функции - и® называются: a. Бесконечно малыми одного порядка при → (в точке), если lim → - ® E 0; b. Эквивалентными бесконечно малыми при → (в точке), если lim → - ® 1 обозначение:-~®при → . Если lim → () 0, то говорят, что - является бесконечной малой более высокого порядка при → (в точке), чем ®, и пишут -²® при → (- равно “² малое” от ® при →). Например, ² при →0. Аналогичные определения имеют место для случаев → 0, → 0, → ∞. Следует иметь в виду, что равенства, содержащие символ “² малое”, являются условными. Например, равенство ² при →0 верно, но ² неверно, поскольку символ ² обозначает не какую-то конкретную функцию, а любую функцию, являющуюся при →0 бесконечно малой более высокого порядка, чем. Таких функций бесконечно много, в частности, любая функция * (где ³ 1) есть ² при →0. Таким образом, равенство ² при →0 означает, что функция принадлежит множеству бесконечно малых функций более высокого порядка при →0, чем. Поэтому “в обратную сторону” это равенство ² неверно: все множество функций ² не сводится к одной функции. Ничего не понятно ☺? Не волнуйтесь, далее мы все рассмотрим на примерах. Но теория в любом случае нужна, иначе моя книга перестает быть математической, и становится непонятно чем. 1.Сравнение бесконечно малых функций. Функция K называется бесконечно малой при → (в точке), если lim → K 0 . Содержание: 1) Сравнение бесконечно малых функций 2) Свойства символа “o малое” 3) Сравнение бесконечно малых функций Высшая математика для чайников. Предел функции 2011 год 56 Рассмотрим несколько примеров, соответствующих данной теме. №1. Верно ли равенство 2 ² при →0? Решение: 2 ² – верно, так как lim → 2 0. Как видите, решение в одну строчку. Давайте его разберем более подробно ☺. Вспомним наше определение! Если lim → () 0, то говорят, что - является бесконечной малой более высокого порядка при → (в точке), чем ®, и пишут -²® при → (- равно “² малое” от ® при →). В нашем случае, мы обозначаем за - 2 . Далее нам нужно от куда-то “выкопать” ®. Посмотрим в определении на слова пишут -²® . Отсюда следует, то, что ® , судя по нашему примеру 2 ². Далее следуем просто определению, т.е. выписываем предел и проверяем, равен ли он нулю или нет. lim → - ® lim → 2 lim → 20 Предел равен нулю, следовательно - 2 является бесконечной малой более высокого порядка при →0 (в точке 0), чем ® , и пишут 2 ²® при →. Так же построим для наглядности наши графики функции. Красный график – это наша «главная» функция - 2 , а зеленый график – это функция ® . По картинке видно, что ближе к нулю функция - 2 стремится к нему быстрее, чем ® . Все! Мы с вами разобрали очень подробно этот пример. Далее все примеры будут идентичными, поэтому так подробно я решение писать не буду. Высшая математика для чайников. Предел функции 2011 год 57 Во всех остальных случаях красный график – это функция- , а зеленый - ® . №2. Верно ли равенство 3² при → 0? Решение: Для начала выпишем функции - и ® . Вот, что у нас получится: - 3,® Теперь смотрим предел: lim → - ® lim → 3 3 0 Предел не равен нулю, следовательно равенство 3² неверно. Но! Так как предел равен константе, то функции 3 и бесконечно малые одного порядка в точке 0. №3. Верно ли равенство b | | ² при →0? Решение: Для начала выпишем функции - и ® . Вот, что у нас получится: - b | | ,® Теперь смотрим предел: lim → - ® lim → b | | lim → b | | b | | ∙ b | | 7 10 8 ∞ 0 Предел не равен нулю, следовательно равенство b | | ² неверно. Высшая математика для чайников. Предел функции 2011 год 58 №4. Верно ли равенство | | ² при →0? Решение: Для начала выпишем функции - и ® . Вот, что у нас получится: - ln | | ,® Теперь смотрим предел: lim → - ® lim → n ln | | olim → 1 ln | | 0 Предел равен нулю, следовательно равенство | | ² верно. №5. Верно ли равенство 1 cos ² при →0? Решение: Для начала выпишем функции - и ® . Вот, что у нас получится: - 1 cos ,® Теперь смотрим предел: lim → - ® lim → 1 cos lim → 2sin ] 2 ^ lim → n sin] 2 ^ 2 o 2 1 ∙ 0 0 Предел равен нулю, следовательно равенство 1 cos² верно. P.S. Решение таких пределов у вас уже Высшая математика для чайников. Предел функции 2011 год 59 не должно вызывать сложностей. Если чувствуете, что не справляетесь, лучше вернуться в главу 1 и 2 и все повторить. Все пределы таких типов у нас уже были. Это, как говорится, база, без которой никуда. Так как примеры все идентичны между собой, сначала решайте их сами, а потом смотрите на решение. Если так делать не будете, то ничему не научитесь!!! №6. Верно ли равенство sin ² при →0? Решение: Для начала выпишем функции - и ® . Вот, что у нас получится: - sin ,® Теперь смотрим предел: lim → - ® lim → sin lim → ! sin # 1 1 Предел не равен нулю, следовательно равенство sin ² неверно. Но! Так как предел равен единицы, то функции sin и эквивалентные бесконечно малые в точке 0. №7. Верно ли равенство ² при →0? Решение: Для начала выпишем функции - и ® . Вот, что у нас получится: - ,® Теперь смотрим предел: lim → - ® lim → 0 Предел равен нулю, следовательно равенство ² верно. Высшая математика для чайников. Предел функции 2011 год 60 №8. Верно ли равенство 1 cos ² при →0? Решение: Для начала выпишем функции - и ® . Вот, что у нас получится: - 1 cos,® Теперь смотрим предел: lim → - ® lim → 1 cos 12 Предел не равен нулю, следовательно равенство 1 cos² неверно. Но! Так как предел равен константе, то функции 1 cos и бесконечно малые одного порядка в точке 0. Высшая математика для чайников. Предел функции 2011 год 61 Пусть - и - две произвольные бесконечно малые при → функции такие, что - ²® и - ²®. Тогда - - ²® при →. Эту теорему можно записать так: ² ® ² ® ² ® . Сформулируем наряду с указанным еще ряд свойств символа “² малое” (всюду имеется ввиду, что - →0 и ® →0 при →). 1. ² ® ² ® ² ® 2. ² ® ² ® ² ® 3. ² E® ² ® ∀E 0 4. E² ® ² ® ∀E 0 5. ² ® ² P ® Q , ´ 2 ∈ µ ,w1,2,…, 1 6. P ² ® Q ² ® ∀ ∈ µ 7. ® ² ® ² ® ∀ ∈ µ 8. +)) ² ® , ´ 2 ∈ µ Обозначим любую бесконечно малую при → функцию символом ² 1 . Тогда свойство 8 будет справедливо также при 1: +)) ²1. 9. o P ∑ c , β , Q o β ,гдеc , числа 10. ² P ² ® Q ² ® 11. ² P ® ² ® Q ² ® 12. -®² - ,-®² ® 13. Если ~®, то - ®²- и - ®²® На сей ноте теория заканчивается и начинается практика. Рекомендую все свойства выучить. В дальнейшем они нам сильно пригодятся. Первая задача будет очень подробно разобрана. Следующие задачи вы должны будете сделать сами, что бы “вникнуть” в эту тему. №1. Используя предел lim -→ .&- - 1 представить функцию sinx в виде ¹ ² P Q при →0,гдеw1илиw2; и некоторыечисла. 2.Свойства символа “ O малое”. Высшая математика для чайников. Предел функции 2011 год 62 Решение: Докажем сначала, что если - и ® бесконечно малые одного порядка при →, т.е. lim → () E 0, то - с® ²® при →. В самом деле, так как lim → - ® E → lim → d - ® E e 0 → lim → - E® ® 0, То по определению символа ²® имеем - E® ²®, или - E® ² ® при →. Пользуясь данным равенством, получаем sinx ² при → 0, Последняя формула называется асимптотической формулой функции sin при →0. Последнее слагаемое в правой части этой формулы ² называется остаточным членом асимптотической формулы. Далее, в последующих примерах, мы не будем доказывать одно и тоже и будем исходить из уже доказанного, т.е. - E® ² ® при →. Поэтому рекомендую прочесть доказательство еще раз, и самое главное, понять его. №2. Используя предел lim -→ /. - представить функцию sinx в виде ¹ ² P Q при →0,гдеw1илиw2; и некоторыечисла. Решение: Используем формулу - E® ² ® при → и получаем: cos 1 12 ² при →0. Последняя формула называется асимптотической формулой функции cos при → 0. Последнее слагаемое в правой части этой формулы ² называется остаточным членом асимптотической формулы. №3. Используя предел lim -→ - 1 представить функцию sinx в виде ¹ ² P Q при →0,гдеw1илиw2; и некоторыечисла. Решение: Высшая математика для чайников. Предел функции 2011 год 63 Используем формулу - E® ² ® при → и получаем: ln1 ² при → 0. Последняя формула называется асимптотической формулой функции ln1 при →0. Последнее слагаемое в правой части этой формулы ² называется остаточным членом асимптотической формулы. №4. Используя предел lim -→ √ - представить функцию sinx в виде ¹ ² P Q при →0,гдеw1илиw2; и некоторыечисла. Решение: Используем формулу - E® ² ® при → и получаем: √ 1 1 1 ² при →0. Последняя формула называется асимптотической формулой функции √ 1 при → 0. Последнее слагаемое в правой части этой формулы ² называется остаточным членом асимптотической формулы. Я думаю, для вас этого будет достаточно. В институте или колледже этому почти не уделяется времени. На сей раз я хотел, что бы вы поняли, откуда берется это “² малое”, и как выводятся асимптотические формулы. Как говорится, немножко теории вам не помешает и, конечно, желательно понимать, что от куда берется. Высшая математика для чайников. Предел функции 2011 год 64 Ранее были уже получены асимптотические формулы для простейших элементарных функций при →0. Запишем эти формулы в виде таблицы. Указанные формулы остаются справедливыми, если в них вместо аргумента подставить, где º » бесконечно малая последовательность, либо, где lim → 0. Например, справедливо представление, вытекающее из первой формулы: sin 1 1 ²! 1 #, где 2 ² ] ^ бесконечно малая последовательность более высокого порядка, чем 2 , т.е. lim → ²] 1 ^ 1 lim → ²! 1 #0. То есть этим мы хотим сказать, что если 2 sin →0, то мы можем применить к синусу асимптотическую формулу. Например, функция 1 является бесконечно малой при → 1, поэтому из третьей формулы получаем равенство ln P 1 Q ² при →1, или ln 1 1 1² при → 1. Вот вам и еще один пример. Используя прошлое равенство и вторую формулу, запишем асимптотическое представление функции cos ln при →1. 1 sin & 6 2 cos 1 2 & 6 3ln 1 & &6 4 1 & ln & 6 0 5 S 1 & & 6 6 1 & 1 & & 6 7 tg & 6 8sh &6 9 ch 1 & 2 & 6 10 th & 6 Высшая математика для чайников. Предел функции 2011 год 65 Функция ln при →1 стремится к нулю, следовательно является бесконечно малой, следовательно можно применить асимптотическую формулу номер три: coslncos 1 ² 1. Функция cos 1 ² 1 при →1 стремится к нулю, следовательно является бесконечно малой, следовательно можно применить асимптотическую формулу номер два: cos lncos 1 ² 11 P 1 ² 1 Q 2 ² ] P 1 ² 1 Q ^. Вот теперь нам и пригодятся свойства “² малое”. Применяем их и получаем: P 1 ² 1 Q 2 1 2 1 ² 1 12 P ² 1 Q 1 2 ² 1 ² 1 1 2 ² 1 . Первое, что мы сделали, это раскрыли числитель – там квадрат суммы. Далее мы просто применяем свойства “² малое”. Если не учили их, посмотрите в таблице, которую я давал ранее. Аналогично, P 1 ² 1 Q 1 ² 1 . Применяем асимптотическое свойство номер 11. Получаем: ² ] P 1 ² 1 Q ^² 1 ² 1 ² 1 . Окончательно получаем cos ln1 1 2 ² 1 при → 1. Так же мы можем записать наше решение и так: lim → cos lnlim → d 1 1 2 ² 1 e . Теперь вы понимаете, зачем нам нужны эти асимптотические формулы! Как бы вы по другому искали этот предел? Запомните, если функция стремится к нулю, мы всегда ее можем заменить асимптотическими формулами. Если же она не стремится к нулю, а, например к какой-нибудь константе или бесконечности, мы не имеем права использовать асимптотические формулы!!! Асимптотические формулы применяются лишь в том случае, когда функция стремится к 0! Высшая математика для чайников. Предел функции 2011 год 66 Давайте посчитаем наш предел: lim → cos lnlim → d 1 1 2 ² 1 e ¦1 1 1 2 §1. Сложно? Нет! Запутанно? Да! Но что же поделаешь, практика здесь определенно нужно. Думаю, через несколько минут вам будет уже все понятно. Переходим к примерам. Так же как и всегда, первый разобран подробно, остальные примеры решайте сначала сами, а потом смотрите решение. №1. Найти предел: lim → ln1 4 sin3 . Решение: Для начала смотрим, можно ли применить асимптотические формулы. Вспоминаем, когда их можно применять? Когда функция стремится к нулю. Проверяем: lim → ln1 4 ln1 0 lim → sin3 sin0 0 Все верно! Значит применяем формулы. В данном случае это ln1 ¼ ~¼,sin¼~¼. Так как пример очень простой, “² малое” мы здесь можем не писать. Если хотите, можете использовать его. Тогда lim → ln1 4 sin3 lim → 43 43 . Как видите, все очень просто. №2. Найти предел: lim → √ 1 1 . Решение: Так как ½ √ 1 1 ¾ →0 и º » →0 при →0, то можем применять асимптотические формулы. √ 1 ~1 3 ,. То есть, Высшая математика для чайников. Предел функции 2011 год 67 lim → √ 1 1 lim → 1 3 1 lim → 3 13 . №3. Найти предел: lim !→ 1 cos1 cos sin . Решение: Так как º 1 cos1 cos » → 0 и º sin » →0 при →0, то можем применять асимптотические формулы. cos ~1 2 ,sin ~. То есть, lim !→ 1 cos1 cos sin lim !→ 1 cos!1 1 2 # sin lim !→ 1 cos 2 Пример упростился, но нам этого недостаточно. Поэтому, так как 2 1 cos ! → 0 и º » → 0 при →0, то можем применять асимптотические формулы. cos ~1 2 . lim !→ 1 cos1 cos sin lim !→ 1 cos!1 1 2 # sin lim !→ 1 cos 2 lim !→ 1 p r 1 ! 2 # 2 s u lim !→ 8 v 18 . №4. Найти предел: lim → √ 1 2 3 1 . Решение: Так как ½√ 1 2 3 1 ¾ → 0 при → 0, то можем применять асимптотические формулы. 1 ~1 . Высшая математика для чайников. Предел функции 2011 год 68 В данном случае, 1/2. Поэтому вот что у нас получится: lim → √ 1 2 3 1 lim → 1 2 3 2 1 12 lim → 2 3 12 lim → 2 3 7 12 ∙ 2 8 1. №5. Найти предел: lim → lnln . Решение: Так как º lnln » →0 при →, то можем применять асимптотические формулы. ln 1 ¼ ~¼. Таким образом получаем: lim → lnln lim → lnln 1 1 lim → ln1 ln 1 lim → ln 1 lim → ln ln lim → ln lim → ln 1 ] 1^ 2 ln 1 ] 1^ →0при → lim → 1 lim → 1 lim → 1 . Скажу честно, что предел не из простейших. Запутаться здесь достаточно легко, поэтому, если вы, “чайник”, взяли этот предел, то вы уже далеко не тот, кем вы были до прочтения этой книги. Вы уже средний студент хорошего института! №6. Найти предел: lim → log 1 2 . Решение: Так как º log 1 » →0 при → 2, то можем применять асимптотические формулы. ln 1 ¼ ~¼. Получаем: lim → log 1 2 lim → log log 2 2 lim → log 2 2 lim → ln/2 ln2 2 1 ln2 lim → ln/2 2 1 ln2 lim → ln 1 ] 2 1^ 2 1 ln2 lim → 2 1 2 1 2 ∙ ln2 lim → 2 2 1 2 ∙ ln2 . №7. Найти предел: Высшая математика для чайников. Предел функции 2011 год 69 lim → sin 1 1 . Решение: Так как º sin 1 » →0 при →1, то можем применять асимптотические формулы. Для синуса у нас есть вот такая формула: sin~. Следовательно, перейдем к новой переменной. Пусть 1. Тогда → 0 при →1. Предел становится равным ¿lim !→ sin 1 1 Далее используем алгебраическое тождество: 1 4 6 4 1 Таким образом находим предел: ¿lim !→ sin 1 1 sin~lim !→ 4 6 4 lim !→ 1 4 6 4 14 . №8. Найти предел: lim → lncos √ 1 1 . Решение: Так как º lncos » →0 и ½√ 1 1 ¾ →0 при →0, то можем применять асимптотические формулы. √ 1 ~1 w ,ln 1 ~. Тогда предел можно записать в виде ¿lim → lncos √ 1 1 lim → ln1 cos 1 !1 3 #1 lim → cos 1 /3 3 lim → 1 cos ¦1 cos ~ 2 §3 lim → 2 32 lim → 32 . №9. Найти предел: lim → sinsintg! 2 # lncos3 . Высшая математика для чайников. Предел функции 2011 год 70 Решение: С виду жуткий примерчик, не правда ли? Не волнуйтесь ☺! Мы всегда все преодолеваем. Давайте так же в этом примере будем использовать “² малое”, для того, что бы наш ответ был уж точно правильным. Запишем асимптотическое разложение числителя, пользуясь асимптотическими формулами для синуса и тангенса и свойствами “² малое”: sinsintg d 2 e = sinsin 2 +² d 2 e ¡ = sin À 2 +² d 2 e +² 2 +² d 2 e ¡ Á = sin 2 +² +² ¡ = sin 2 +² ¡ = 2 +² . Здесь мы пользовались тем, что ² d +² ] ^ e = ²() и ² +² = ²(). Выведем теперь асимптотическое разложение знаменателя, используя асимптотические формулы для косинуса и логарифма: lncos3 = ln1 − 3 2 +² 3 ¡ = lnn1 +− 9 2 +² ¡o =− 9 2 +² ¡ +² − 9 2 +² ¡ = − 9 2 +² +² = − 9 2 +² . Здесь мы воспользовались тем, что ² 3 = ² ,² − 9 2 +² ¡ = ² ,² +² = ² . Таким образом данный предел равен lim → sinsintg! 2 # lncos 3 = lim → 2 +²() − 9 2 +²() = lim → 12 + ²() − 92 + ²() = 12 +lim → ²() − 92 + lim → ²() = − 19 . Здесь мы воспользовались тем, что, по определению символа “² малое” lim → ² = 0. Высшая математика для чайников. Предел функции 2011 год 71 От автора: Должен сказать, что если вы все-таки достигли этой страницы, то вы уже далеко не чайник! Вы уже вполне образованный человек, который хорошо разбирается в пределах функций. Я попытался объяснить вам данную тему как можно более понятно. Надеюсь, мне это удалось сделать. Далее вас будет ждать большая и очень важная тема. Это – производные и дифференциалы. Потом, в моих планах стоит тема «неопределенный интеграл», далее – «основные теоремы о непрерывных и дифференцируемых функциях». Но это все пока что в планах. Данную часть я написал и весьма доволен этим. Наверняка, в книге, присутствуют как грамматические ошибки, так и математические (потеря знака). Прошу об этом писать мне на почту… А сейчас можете смело переходить к дополнительным главам ☺. Удачи! С Уважением, Ваш Виосагмир И.А. [email protected] Высшая математика для чайников. Предел функции 2011 год 72 Глава 4. Дополнительные методы. Давайте рассмотрим дополнительные методы, при помощи которых мы можем считать наши пределы. В некоторых случаях данными методами намного проще воспользоваться, чем теми, что мы с вами уже прошли. Но должен предупредить, что здесь вы должны знать как можно и нужно дифференцировать функцию. Сейчас я на этом останавливаться не буду, так как данная тема подробно рассматривается в моей второй книге. Итак, чем же этот метод Лопиталя такой особенный? А особенный он тем, что может раскрывать неопределенности вида V 0 0 v W и ∞ ∞ ⁄ . Если вспомнить, то мы уже много прошли способов для раскрытия различных неопределенностей, но бывают такие случаи, когда сложно ее раскрывать, ну, или по крайней мере неудобно. Но опять же, правило Лопиталя применимо не во всех случаях. Общая формулировка выглядит так: При некоторых условиях предел отношения функций равен пределу отношения их производных. Давайте рассмотрим эти условия ☺. 1. lim → lim → O0или∞ 2. и O дифференцируемы в проколотой окрестности 3. O 0 0 в проколотой окрестности 4. существует lim → ′ O′ à Тогда, если выполняются условия 1 2 3 4 → lim → O lim → ′ O′ . Заметьте, что →, а не к какой-то там бесконечности или вообще нулю. Нам важно то, что предел этих функций должен быть равен бесконечности или нулю! Многие по началу путаются с этим, поэтому не пропускайте это мимо ушей ☺. Содержание: 1) Правило Лопиталя 2) Разложение в ряд Тейлора. Часть 1 3) Разложение в ряд Тейлора. Часть 2 1.Правило Лопиталя Высшая математика для чайников. Предел функции 2011 год 73 Думаю, больше теории здесь давать не нужно. Моя книга больше нацелена именно на практику, поэтому мы сейчас же к ней и переходим. №1. Найти предел lim → +5 3 . Решение: Для начала выпишем наши функции () и O() = +5,O = 3 Теперь проверяем наши условия 1. lim → () = lim → +5 = 0,lim → O() = lim → 3 = 0 →! 2. () и O() дифференцируемы в проколотой окрестности. Т.е. можно взять производную от этих функций в точке = 0 −! 3. O 0 = 3 ≠ 0 в проколотой окрестности 0 −! 4. существует lim → ′() O′() à = lim → 2 +5 3 v −! Когда привыкните, то не будете тратить на проверку свое драгоценное время. Я вам показал как это делать. Теперь, я буду проверять лишь первый пункт. Вам же напутствие – проверяйте каждый пункт! Потому что всякое может быть. lim → +5 3 = 7 00 − ÄÅ 8 = lim → +5 0 3 0 = lim → 2 +5 3 = 7 0 +5 3 − 8 = 53 Вот это лучшая запись решения данного примера! 1 − определяем на неопределенность; 2 − расписываем производные; 3 − считаем производные и одновременно смотрим, стремится ли () и O() к 0; 4 − определяем на неопределенность; 5 − пишем ответ. Легко? Да! Но нужна практика, чтобы не запутаться. №2. Найти предел lim → +4 +7 +3 Решение: = +4 +7 → ∞ при →∞ и O = +3 → ∞ при →∞. Следовательно, можем применить правило Лопиталя ☺. lim → +4 +7 +3 = ∞∞ = lim → +4 +7 0 +3 0 = lim → 3 +8 +7 3 +6 = ∞∞ = lim → 3 +8 +7 ′ 3 +6 ′ = lim → 6 +8 6 +6 = ∞∞ = lim → 6 +8 ′ 6 +6 ′ = lim → 66 = 7 66 8 = 1 Высшая математика для чайников. Предел функции 2011 год 74 Здесь нам пришлось применять правило Лопиталя 3 раза, от того, что определенность никак не хотела уходить! Прежде чем начинать дифференцировать вы должны проверить условия на функции. Здесь вы проверяли условия 4 раза! Они обозначены красным цветом – шаги, на которых вы проверяете условия прежде чем перейти к следующему шагу. Должен сказать, что вы уже наверное поняли, что этот способ для данного примера явно не является оптимальным. Здесь лучше воспользоваться тем, чем мы занимались половину этой книги – вынести из числителя и знаменателя. lim → +4 +7 +3 = lim → ]1 + 4 + 7 ^ ]1 + 3 ^ = lim → 1 + 4 + 7 1 + 3 = 7 1 +0 +0 1 +0 8 = 1 А можно еще и так сделать: lim → +4 +7 +3 = ∞∞ = lim → +4 +7 0 +3 0 = lim → 3 +8 +7 3 +6 = lim → ]3 + 8 + 7 ^ ]3 + 6 ^ = lim → 3 + 8 + 7 3 + 6 = 7 1 +0 +0 3 +0 8 = 33 = 1 То есть, на первом шаге мы проверяем на неопределенность и применяем правило Лопиталя, но тут же догадываемся, что нужно будет так делать еще два раза. Что бы сэкономить наше время, мы выносим высшую степень в числитель, дабы у нас получились бесконечно малые функции. Почему я так трачу много времени на это? Я хочу что бы вы во всем разобрались и поняли, что различные способы можно смешивать между собой! При этом не надо забывать об условиях в каждом таком способе. №3. Найти предел lim → ln 1 +2lnsin Решение: Вот именно для таких случаев у нас и есть правило Лопиталя. А как решить по-другому? Ну, разве что какой-нибудь заменой. Так как все условия выполняются (проверьте их сами), то мы можем применить правило Лопиталя. lim → ln 1 +2lnsin = ∞∞ = lim → ln 0 1 +2lnsin 0 = lim → 1 2 ∙ cos sin = lim → sin 2 cos А не было ли у нас ранее подобного примера ☺? По-моему это налицо первый замечательный предел. Запишем его покрасивее: lim → sin 2 cos = 12 ∙ lim → 7 ! sin #∙ 1 cos 8 = 12 ∙ 1 ∙ lim → 1 cos = 12 Высшая математика для чайников. Предел функции 2011 год 75 Поэтому, lim → ln 1 +2lnsin = 12 . Видите, правило Лопиталя помогает нам дойти до определенного места. А потом мы применяем то, что прошли с вами ранее ☺. Двигаемся дальше… №4. Найти предел lim → 1 −cos 4 Решение: Так как все условия выполняются (проверьте их сами), то мы можем применить правило Лопиталя. lim → 1 −cos 4 = 7 00 8 = lim → 1 −cos 4 0 0 = lim → 4sin4 2 = 7 00 8 = lim → (4sin4)′ (2)′ = lim → 16cos 4 2 = 8 Здесь мы применили правило Лопиталя два раза. Кстати, здесь можно было бы решить и при помощи первого замечательного предела, после первого применения правила Лопиталя. У нас было бы вот так lim → 4sin4 2 = lim → ! sin4 4 #∙ 8 = 8 №5. Найти предел lim → ln Решение: Дроби, как видите, у нас здесь нет. Поэтому мы не можем применять правило Лопиталя. Но ведь мы смекалистые, поэтому мы сейчас сами сделаем дробь ☺. ln = ln 1 v Теперь все верно! Проверьте сами условия и убедитесь в том, что мы имеем права применять правило Лопиталя. lim → ln = 0 ∙ ∞ = lim → ln 1 v = 7 00 8 = lim → ln ′ P 1 v Q ′ = lim → 1 − 1 = −lim → = 0 = 0 №6. Найти предел lim → ! 1 −1 − 1 ln # Высшая математика для чайников. Предел функции 2011 год 76 Решение: Здесь так же как и в предыдущем примере нужно сделать дробь. Надеюсь, вы знаете, как складывать дроби с разными знаменателями ☺. 1 −1 − 1 ln = ln − +1 −1 ln Теперь все верно! Проверьте сами условия и убедитесь в том, что мы имеем права применять правило Лопиталя. lim → ! 1 −1 − 1 ln #= ∞−∞ = lim → ln − +1 −1 ln = 7 00 8 = lim → (ln − +1)′ (−1 ln)′ = lim → 1 −1 ln + −1 = lim → 1 − ln + −1 = 7 00 8 = lim → (1 −)′ (ln + −1)′ = lim → −1 1 +ln +1 = 7 − 12 8 = − 12 Здесь мы изначально перешли к дроби, потом применили правило Лопиталя два раза подряд. №7. Найти предел lim → 1 + Решение: Здесь можно попробовать перейти ко второму замечательному пределу. Мы же попробуем применить правило Тейлора. Для этого нужно сделать дробь. Сделаем достаточно хитро – обозначим 1 + за. То есть, 1 + = →ln = 1 ∙ ln 1 + = ln 1 + Теперь используем очень полезное в данный момент свойство: Так как Ä функция непрерывная, то lnlim → = lim → ln Готов поспорить, половина из вас ничего не поняла ☺. Короче, в данном примере мы переходим от одной функции к другой, не забыв при этом поменять пределы. º → 0при | →∞приln » Верно? Да! Вспомните график логарифма. Соответственно, поменяв пределы, мы начинаем искать предел, пользуясь правилом Лопиталя. lim → ln = lim → ln 1 + = ∞∞ = lim → ln 1 + ′ ′ = lim → 2 1 + = ∞∞ = lim → (2)′ 1 + ′ = lim → 2 2 = 0 Теперь не забываем перейти к обратным переделам! Т.е. у нас получается Высшая математика для чайников. Предел функции 2011 год 77 lim → = илиlim → 1 + = 1 Интересный примерчик ☺? Самое главное, что бы вы поняли то, что в один и тот же пример можно решить различными способами, а не только одним. №8. Найти предел lim → −2arctg ln Решение: Мы не можем применять правило Лопиталя, так как нет дроби. Поэтому, мы ее делаем −2arctg ln = −2arctg 1 ln Вы проверяете 4 свойства и понимаете, что можно применить правило Лопиталя. lim → −2arctg ln = lim → −2arctg 1 ln = () 7 00 8 = lim → −2arctg ′ ] 1 ln ^ ′ = lim → − 2 1 + − 1 ln = lim → 2 ln 1 + = () ∞∞ = lim → (2 ln)′ (1 +)′ = lim → 2ln +4ln 2 = () ∞∞ = lim → (2ln +4ln)′ (2)′ = lim → 4 ∙ ln + 4 2 = 2 lim → ln +1 = () ∞∞ = lim → (ln +1)′ ′ = lim → 1 v 1 = lim → 1 = 0 Мы здесь использовали целых четыре правила Лопиталя! С виду решение, конечно, он красивое ☺. Хочу вам сказать, что такие примерчики далеко не в каждом вузе решают. Я же хочу, что бы вы такие решали! И не были, так сказать, “чайниками”. №9. Найти предел lim → arcsin 1 Решение: Здесь тоже немного хитро ☺. Нужно воспользоваться свойством логарифма arcsin 1 = 1∙23/.& = 23/.& /1 Как мы это сделали? Все просто. Есть такая формула: = Мы ей просто пользуемся и получаем Высшая математика для чайников. Предел функции 2011 год 78 arcsin 1 = 4 23/.& 5 = 1∙23/.& = 23/.& /1 То есть, мы все можем записать так: lim → arcsin 1 = &" → 6 1∙23/.& 7 = &" → 9 23/.& /1: = ; < = &" → 9 23/.& 0 (/1)0: = &" → 9 .& √ ∙23/.& : = &" → 9 ∙ : = = 1 Вот этот пример уже не шутка. Это полноценный, выше среднего уровня, пример! №10. Найти предел lim → ctg Решение: Здесь делаем то же самое, что и в предыдущем примере. Таким образом у нас получается lim → ctg = &" → /1 = ; < = &" → /1 0 ()0 = &" → .& ∙/1 = &" → .&∙/. = = 1 №11. Найти предел lim → −sin +sin Решение: Здесь нельзя применять правило Лопиталя! Проверьте все условия и поймите, что я говорю все верно ☺! Здесь нужно вычислять предел вот так lim → −sin +sin = lim → ]1 − sin ^ ]1 + sin ^ = lim → 1 − sin 1 + sin = 1 №12. Найти предел lim → ! sin # Решение: lim → ! sin # = &" → = .& >∙ = &" → .& = ; < = &" → .& ()0 = &" → /. = ; < = &" → = Высшая математика для чайников. Предел функции 2011 год 79 На этом мы заканчиваем правило Лопиталя. Запомните одну важную вещь! Не стоит применять это правило езде и вся. Сначала определите, а нужно ли вообще его здесь применять? Когда у вас логарифмы, синусы, корни, то оно может помочь. Но если у вас простые выражения, оно может только затруднить вашу работу. Так что никуда не спешите ☺. Высшая математика для чайников. Предел функции 2011 год 80 В данном разделе мы рассмотрим предел функции вида O ⁄ . Что такое разложение ряда Тейлора и все его подробности я рассказывать не буду, так как это все написано в моей второй книге. В данном разделе я на примерах объясню принцип данной работы. В таблице представлены основные разложения по формуле Тейлора при условии, что →0. Их можно не запоминать, просто распечатайте и пользуйтесь ими. А сейчас мы разберем метод Тейлора на конкретных примерах. Я называю данные примеры crash-примерами. Сейчас поймете, почему именно такое название ☺. №1. Найти предел lim → cos arctg ln 1 Решение: Так как в знаменателе одна функция, то представим ее формулой Маклорена до остаточного члена ² , то есть sin 6 & 120 & 6 cos 1 2 & 24 & 6 7 Y & 6 & 120 & 6 " Y 1 & 2 & 24 & 6 tg & 3 & 2 15 & 6 8 Y 3 & 2 15 & 6 arcsin & 6 & 3 40 & 6 arctg 3 & 5 & 6 ln 1 & 2 & 3 4 & 6 ln 1 2 3 4 & 6 ln > & Z 1 & E 6 & 3 40 & 6 1 1 & 1 & & & 6 1 1 1 & & & & & 6 √ 1 & 1 & 2 8 & 16 & 6 2.Разложение в ряд Тейлора. Часть 1 Высшая математика для чайников. Предел функции 2011 год 81 O = − ∙ +² = − +²() Знаменатель дроби легко представить в виде ряда Маклорена. Нам все члены не нужны, поэтому мы берем самый первый, ненулевой. Теперь рассмотрим числитель. Так как знаменатель мы разложили до остаточного члена ² , то и числитель мы должны раскладывать точно до такого же остаточного члена. cos = 1 − 2 +² → ∙ cos = − 2 +² arctg = − 3 +² Как видите, мы раскладываем cos до остаточного члена ² , так как уже знаем, что cos мы помножим на, и он нам даст остаточный член ² . В итоге, вот наш разложенный числитель: = − 2 +² − − 3 +² ¡ = − 6 +² Тогда lim → () O() = lim → − 6 +² − +²() = 16 Вот мы с вами и посчитали первый предел ☺. Запутанно? Да. Но при помощи рядов Тейлора можно считать очень сложные и “непроходимые” пределы. Зная как это делать, вы потратите достаточно времени на поиск предела, но зато вы его в конечном счете посчитаете! Вы останетесь в выигрыше ☺. №2. Найти предел lim → sin] 1 − ^ +ln(1 −) − 2 tg(Åℎ) −arctg Решение: Для начала рассмотрим знаменатель и попробуем найти функцию O(). Для этого разложим наши функции tg(Åℎ) и arctg. Теперь возникает вопрос, а до какого остаточного члена нам раскладывать? Ну, для начала, давайте попробуем до ²(). Åℎ = +²() O = +² ,где = Åℎ Теперь подставим и найдем O(Åℎ) O Åℎ = +² +² P +² Q = +²() Высшая математика для чайников. Предел функции 2011 год 82 Но давайте посмотрим на числитель. Там уже остаточный член при разложении будет больше чем ²(). Как я уже говорил, остаточный член везде должен быть одинаковым. Поэтому нам придется раскладывать до ² . Åℎ = + 3! +² O = + 3 +² ,где = Åℎ Теперь подставим и найдем O(Åℎ) O Åℎ = + 3 +² = + 3! +² ¡ + d + 3! +² e 3 +² + 3! +² ¡ Теперь обратим внимание на второе слагаемое, т.е. на d + 3! +² e 3 Если мы раскроем скобки в числителе, то получится + 2 + % 4 + 19 +² Но! Нам ² не нужно, нам нужно ² , как мы и договорились раньше. Поэтому мы можем избавиться от членов 2 + % 4 + 19 Потому что они дают нам ² . Повторяю еще раз, если мы решили что в нашем примере остаточный член будет представлен в виде ² , значит он должен быть в каждом слагаемом именно такой и не иначе! Соответственно, мы можем написать так: O Åℎ = + 3 +² = + 3! +² ¡ + P +² Q 3 +² = + 2 +² Разложим второе слагаемое в знаменателе. Оно у нас уже есть в таблице arctg = − 3 +² Таким образом, функция знаменателя O() раскладывается так O = + 2 +² ¡ − − 3 +² ¡ = 5 6 +² Высшая математика для чайников. Предел функции 2011 год 83 Теперь перейдем к числителю. Для начала рассмотрим 1 − У нас есть формула для вида дроби 1 1 − Мы сделаем хитро. Разложим дробь до остаточного члена ² , так как при умножении потом на у нас получится оценка ² . А она как раз нам и нужна! 1 1 − = 1 + + +² Тогда, при умножении на у нас получится 1 − = P 1 + + +² Q = + + +² Разложим sin, где = 1 − v . Эта формула нам также известна (в таблице). sin = − 3! +² Здесь мы разложили так же до ² , так как никаких умножений на sin у нас нет. Теперь подставим все под и получим sin] 1 − ^ = P + + +² Q − P + + +² Q 3! +² ] P + + +² Q ^ Теперь рассмотрим нашу дробь P + + +² Q 3! Обратите внимание на числитель. Если мы раскроем скобки, то наша оценка значительно увеличится, а нам этого не нужна. Нам нужно, что бы оценка оставалась ² . Что делать? Избавляться от остальных членов! Таким образом дробь примет несколько другой вид P +² Q 3! Конечно, если хотите, то можете раскрыть все скобки P + + +² Q ,Э а потом выкинуть все, степень которых будет больше 3. Но вы замучаетесь это делать, поэтому выкидывайте их сразу! Итого, вот что у нас получится Высшая математика для чайников. Предел функции 2011 год 84 sin] 1 − ^ = P + + +² Q − P +² Q 3! +² = + + +² − +² 3! = + + 5 6 +² Рассмотрим второе слагаемое в числителе, то есть ln(1 −) Слава богу, его разложение у нас уже есть в таблице ln(1 −) = − − 2 − 3 +² Итого, мы можем записать нашу () функцию = + + 5 6 +² ¡ +− − 2 − 3 +² ¡ − 2 = 2 +² Теперь у нас есть разложенные функции () и O(). Мы можем найти наш предел lim → () O() = lim → 2 +² 5 6 +² = 35 Мы нашли предел! Хочу сказать, что это высший уровень! Это не “чайник” и не “среднячок”. Это мега-студент, который может многое. Господа, повышайте свою самооценку и чувствуйте себя выше других, решая такие примеры ☺. Лично я искренне надеюсь, что вы все поймете (а может и уже поняли) все что я рассказываю вам. Ну, что!? Идем дальше покорять вершины математики ☺! №3. lim → O P Q − ln Eℎ arctg(cos) −tg Решение: Красота, не правда ли ☺? Ничего, справились с предыдущим, покорим и этот! Будем представлять до точности ² , как и в предыдущих номерах. Попробуем вывести функцию O(). Для этого рассмотрим cos (его разложение нам известно) cos = 1 − 2 +² Остаточный член представлен в виде ² , так как на cos мы умножаем, который нам дает нашу лучшую оценку ² . Высшая математика для чайников. Предел функции 2011 год 85 cos = 1 − 2 +² ¡ = − 2 +² Теперь разложим arctg , где = cos (так же по таблице) EO = − 3 +² Тогда мы можем разложить arctg(cos) arctg(cos) = − 2 +² ¡ − d − 2 +² e 3 +² n − 2 +² ¡ o Если мы обратим внимание на числитель второй дроби, то есть на − 2 +² ¡ , то мы сразу же обратим внимание на то, что при раскрытии скобок мы никак не получим ² . Степень у будет значительно выше. Поэтому мы избавляемся от ненужных нам членов и получаем arctg(cos) = − 2 +² ¡ − +² 3 +² = − 5 6 +² Нам осталось разложить последнее слагаемое в знаменателе O = + 3 +² Таким образом мы собрали все нужные нам данные для того, чтобы найти функцию O(). O = arctg(cos) −tg = − 5 6 +² ¡ − + 3 +² ¡ = − 7 6 +²() Отлично! Мы смогли представить знаменатель с точностью до ² . Поэтому мы смело можем переходить к числителю. Нам нужно разложить O P Q − ln Eℎ Как вы вероятно уже поняли, мы начинаем с внутренних функций. Поэтому, для начала разложим! , где = − . ! = 1 + +²() Высшая математика для чайников. Предел функции 2011 год 86 Как видите, мы раскладываем с точностью до ²(), так как даст нам точность ² , а − ² . = 1 − +² = P 1 − +² Q = − +² Теперь разложим O, где = . O = + 3 +² Подставим и получим O P Q = P − +² Q + P − +² Q 3 +² ] P − +² Q ^ Рассмотрим числитель второй дроби P − +² Q Если мы раскроем скобки, то у нас уже не будет точности ² , поэтому от других членов мы просто избавляемся. O P Q = P − +² Q + P +² Q 3 +² = − 2 3 +² Отлично! Одно слагаемое мы смогли представить. Теперь рассмотрим второе ln Eℎ Здесь есть тоже своя хитрость. Так как мы делим на, то числитель нам нужно представить с точностью до ² , что бы при делении точность всей дроби была ² . ln(Eℎ) = 2ln(Eℎ) Здесь мы применили просто свойство логарифма. Eℎ = 1 + 2 + 24 +² Теперь разложим ln(+1),где = Eℎ −1. Мы раскладываем ln(+1), так как у нас нет формул разложения для ln. = Eℎ −1 − этим мы компенсируем нашу единичку. Высшая математика для чайников. Предел функции 2011 год 87 ln(+1) = − 2 + 3 − 4 +² = 1 + 2 + 24 +² ¡ − d 1 + 2 + 24 +² e 2 + d 1 + 2 + 24 +² e 3 − d 1 + 2 + 24 +² e 4 +² n1 + 2 + 24 +² ¡ o Ну, что же. Здесь мы должны откинуть все члены, что бы оценка не увеличивалась, а так же и оставалась на уровне ² . Вот что у нас получается в конечном итоге ln(Eℎ) = 2ln(Eℎ) = 2ln1 + 2 + 24 +² ¡ = 2 p q q r 2 + 24 +² − d 2 +² e 2 +² s t t u = 2 2 + 24 +² − 8 +² ¡ = − 6 +² Таким образом, мы можем расписать нашу функцию () = − 2 3 +² ¡ − 1 − 6 +² ¡ = − 2 +² Отсюда можно найти предел lim → () O() = lim → − 2 +² − 7 6 +²() = 37 Высшая математика для чайников. Предел функции 2011 год 88 В данной теме мы рассмотрим предел функции вида? . Так же как и в прошлом разделе, рассмотрим все на примерах. №1. Найти предел функции lim → d √1 cos e Решение: Распишем разложение функции. Это сделать легко, так как все разложения у нас есть в таблице. √1 cos 1 12 18 ² 1 12 1 24 ² d 1 12 18 ² eÆ 1 d 12 1 24 ² e 2 ² ¡ ² Ç 1 2 8 ² ¡1 2 5 24 ² ¡1 6 ² Отсюда легко найти и предел lim → ? lim → 1 6 ² ¡ / Как считать второй замечательный предел мы с вами уже проходили, поэтому я не буду тратить сейчас на это времени. 3.Разложение в ряд Тейлора. Часть 2

Похожие публикации